You are on page 1of 113

Login ID:8FA03376/Student Name:ANSHULGARG/Overall Score:28

Student ScoreCard
Score:0

Percentile: NA

Test of Logical Reasoning

Score:22

Percentile: NA

Test of Quantitative Aptitude

Score:0

Percentile: NA

Test of VA & RC

Score:6

Percentile: NA

Overall: Overall

Score:28

Percentile: NA

ite
d

Test of Abstract Reasoning

3)

4)

5)

rc

2)

ou

es

1)

es

Pr

iv
at
e

Li
m

Question: 1
Choose the option which will replace the missing figure in the series.

ng

Explanation:
In first step one element is rotated by 180starting from the top element. In second step 2 elements are rotated, in third step 3 elements.
Therefore, in fourth step 4 elements should be rotated by 180. This is followed in choice 3. Hence, [3].

IM

Le

ar

ni

Question: 2
Choose the option which will replace the missing figure in the series.

1)

2)

IMS Learning Resources Pvt.Ltd.,Mumbai.All copyrights to this material vestswith IMS Learning Resources Pvt.Ltd.
No part of this materials either in part oras a whole shall be copied,printed,electronically reproduced,sold or distributed without the written
consent of IMS Learing Resources Pvt.Ltd.and any such violation would entail initiation of suitable legal proceedings.

Copyright

Login ID:8FA03376/Student Name:ANSHULGARG/Overall Score:28

3)

4)

5)

Explanation:
In the successive steps, 2, 3, 4 and 5 arrows from the left end rotate by 180oand the circle moves by 1, 2, 3 and 4 steps towards the right.
Thus, the missing figure is option 3. Hence, [3].

3)

4)

5)

es

2)

rc

ou

1)

Pr

iv
at
e

Li
m

ite
d

Question: 3
Choose the option which will replace the missing figure in the series.

es

Explanation:
In successive steps, the dot moves by 1, 2, 3 and 4 places in the anticlockwise direction while the arrow rotates by 90 in the anticlockwise
direction in one step and rotates 180 in the next step. Thus, the missing figure is option [2]. Hence, [2].

IM

Le

ar

ni

ng

Question: 4
Choose the option which will replace the missing figure in the series.

1)

2)

3)

IMS Learning Resources Pvt.Ltd.,Mumbai.All copyrights to this material vestswith IMS Learning Resources Pvt.Ltd.
No part of this materials either in part oras a whole shall be copied,printed,electronically reproduced,sold or distributed without the written
consent of IMS Learing Resources Pvt.Ltd.and any such violation would entail initiation of suitable legal proceedings.

Copyright

Login ID:8FA03376/Student Name:ANSHULGARG/Overall Score:28

4)

5)

Explanation:

3)

4)

5)

rc

2)

ou

es

1)

es

Pr

iv
at
e

Li
m

ite
d

Question: 5
Choose the option which will replace the missing figure in the series.

ni

ng

Explanation:

IM

Le

ar

Question: 6
Each of the following questions consists of unmarked figures followed by five figures marked 1, 2, 3, 4 and 5. Select a figure from the
marked figures which will continue the series established by the unmarked figures.

IMS Learning Resources Pvt.Ltd.,Mumbai.All copyrights to this material vestswith IMS Learning Resources Pvt.Ltd.
No part of this materials either in part oras a whole shall be copied,printed,electronically reproduced,sold or distributed without the written
consent of IMS Learing Resources Pvt.Ltd.and any such violation would entail initiation of suitable legal proceedings.

Copyright

2)

3)

4)

5)

Li
m

iv
at
e

1)

ite
d

Login ID:8FA03376/Student Name:ANSHULGARG/Overall Score:28

es

Pr

Explanation:
In fig (2) and (3) a new arrow with a differrent arrowhead (pointing in the direction opposite of the arrow above it) is added. In fig (4) there
is a repetition of arrow in fig (1), following the same principle of pointing in opposite directions. Therefore the arrow introduced in fig (3) will
be repeated in the answer figure but pointing in the opposite direction. Looking at the options, option (4) is the best option. Hence, [4].

2)

IM

1)

Le

ar

ni

ng

es

ou

rc

Question: 7
Each of the following questions consists of unmarked figures followed by five figures marked 1, 2, 3, 4 and 5. Select a figure from the
marked figures which will continue the series established by the unmarked figures.

3)

4)

5)

Explanation:
On the right hand side of the line, in alternate steps, one upper and lower line foming the arrow head bend such that they form a 90oangle
with the main line. Using this logic option (1) is a logical continuation of the series. Hence, [1].

IMS Learning Resources Pvt.Ltd.,Mumbai.All copyrights to this material vestswith IMS Learning Resources Pvt.Ltd.
No part of this materials either in part oras a whole shall be copied,printed,electronically reproduced,sold or distributed without the written
consent of IMS Learing Resources Pvt.Ltd.and any such violation would entail initiation of suitable legal proceedings.

Copyright

Login ID:8FA03376/Student Name:ANSHULGARG/Overall Score:28

2)

3)

4)

5)

iv
at
e

Pr

1)

Li
m

ite
d

Question: 8
Each of the following questions consists of unmarked figures followed by five figures marked 1, 2, 3, 4 and 5. Select a figure from the
marked figures which will continue the series established by the unmarked figures.

rc

es

Explanation:
In alternate steps starting from the second step, the outermost or main object rotates by 90oin the clockwise direction. The elements inside
the main object in every step rotate by 90oin the clockwise direction. Thus, if the series is continued in the given manner, option [4] should
come next in the series. Hence, [4].

1)

2)

Le

ar

ni

ng

es

ou

Question: 9
Each of the following questions consists of unmarked figures followed by five figures marked 1, 2, 3, 4 and 5. Select a figure from the
marked figures which will continue the series established by the unmarked figures.

IM

3)

4)

5)

Explanation:
The elements shift by 1 and a half positions in the clockwise direction and half a position in the clockwise direction in alternate steps. Also,
in every step, the elements reverse their shading. Thus, option 4 should come next in the series. Hence, [4].

IMS Learning Resources Pvt.Ltd.,Mumbai.All copyrights to this material vestswith IMS Learning Resources Pvt.Ltd.
No part of this materials either in part oras a whole shall be copied,printed,electronically reproduced,sold or distributed without the written
consent of IMS Learing Resources Pvt.Ltd.and any such violation would entail initiation of suitable legal proceedings.

Copyright

Login ID:8FA03376/Student Name:ANSHULGARG/Overall Score:28

2)

3)

4)

5)

iv
at
e

Pr

1)

Li
m

ite
d

Question: 10
Each of the following questions consists of unmarked figures followed by five figures marked 1, 2, 3, 4 and 5. Select a figure from the
marked figures which will continue the series established by the unmarked figures.

ng

es

ou

rc

es

Explanation:

IM

Le

ar

ni

Question: 11
Each of the following questions consists of unmarked figures followed by five figures marked 1, 2, 3, 4 and 5. Select a figure from the
marked figures which will continue the series established by the unmarked figures.

1)

2)

IMS Learning Resources Pvt.Ltd.,Mumbai.All copyrights to this material vestswith IMS Learning Resources Pvt.Ltd.
No part of this materials either in part oras a whole shall be copied,printed,electronically reproduced,sold or distributed without the written
consent of IMS Learing Resources Pvt.Ltd.and any such violation would entail initiation of suitable legal proceedings.

Copyright

Login ID:8FA03376/Student Name:ANSHULGARG/Overall Score:28

3)

4)

5)

Explanation:
The shorter line alternately changes its position and then moves one step downwards. Whereas the line with the circle in each figure
changes its position (i.e., it alternately moves to the front and back of the circle) and moves along with the circle one step upwards till the 4
thfigure, then in 5thfigure, it returns to the bottom again and continues its movement in the same manner. Hence, [5].

3)

4)

5)

2)

ng

ni

1)

es

ou

rc

es

Pr

iv
at
e

Li
m

ite
d

Question: 12
In each of the following questions, there is a set of five figures labelled (i), (ii), (iii), (iv), and (v) called the Problem Set followed by a set of
five other figures labelled (1), (2), (3), (4), and (5) called the Answer Set. One of the figures contains a question mark. Select a suitable
figure from the Answer Set which will substitute this question mark so that a series is formed by the figures (i), (ii), (iii), (iv), and (v) taken in
order. The number of the selected figure is the answer.

Le

ar

Explanation:
Each elements rotates by 90 in the clockwise direction. Hence, [1].

IM

Question: 13
In each of the questions given below ,which one of the five answer figures (from 1, 2, 3, 4 and 5) should come after the problem figures
[which are in the sequence (i)-(ii)-(iii)-(iv)-(v)], if the sequence were continued?

IMS Learning Resources Pvt.Ltd.,Mumbai.All copyrights to this material vestswith IMS Learning Resources Pvt.Ltd.
No part of this materials either in part oras a whole shall be copied,printed,electronically reproduced,sold or distributed without the written
consent of IMS Learing Resources Pvt.Ltd.and any such violation would entail initiation of suitable legal proceedings.

Copyright

2)

3)

4)

5)

iv
at
e

Pr

1)

Li
m

ite
d

Login ID:8FA03376/Student Name:ANSHULGARG/Overall Score:28

es

ou

rc

es

Explanation:

IM

Le

ar

ni

ng

Question: 14
In each of the questions given below ,which one of the five answer figures (from 1, 2, 3, 4 and 5) should come after the problem figures
[which are in the sequence (i)-(ii)-(iii)-(iv)-(v)], if the sequence were continued?

1)

IMS Learning Resources Pvt.Ltd.,Mumbai.All copyrights to this material vestswith IMS Learning Resources Pvt.Ltd.
No part of this materials either in part oras a whole shall be copied,printed,electronically reproduced,sold or distributed without the written
consent of IMS Learing Resources Pvt.Ltd.and any such violation would entail initiation of suitable legal proceedings.

Copyright

Login ID:8FA03376/Student Name:ANSHULGARG/Overall Score:28

2)

3)

4)

5)

ite
d

Explanation:
'L' object rotates 90 clockwise at each step in alternate steps number of 'L''s increase by 1. Elements at bottom should interchange
position at every step. Element at the bottom left is replaced. Hence, [5].

3)

4)

5)

es

2)

ng

ni

1)

ou

rc

es

Pr

iv
at
e

Li
m

Question: 15
In each of the following questions, there is a set of five figures labelled (i), (ii), (iii), (iv) & (v) called the Problem set followed by a set of five
other figures labelled 1, 2, 3, 4 and 5 called the Answer set. Select a suitable figure from the Answer set which will substitute this question
mark so that a series is formed by the figures (i), (ii), (iii), (iv), and (v) taken in order. The number of the selected figure is the answer.

ar

Explanation:
The number of elements increase by 1 in each step. Hence, [5].

IM

Le

Question: 16
In each of the following problems, a related pair of figures is given followed by five numbered pairs of figures (1), (2), (3), (4) & (5). Select
a pair that has a relationship similar to that in the original pair.

IMS Learning Resources Pvt.Ltd.,Mumbai.All copyrights to this material vestswith IMS Learning Resources Pvt.Ltd.
No part of this materials either in part oras a whole shall be copied,printed,electronically reproduced,sold or distributed without the written
consent of IMS Learing Resources Pvt.Ltd.and any such violation would entail initiation of suitable legal proceedings.

Copyright

2)

3)

4)

5)

iv
at
e

Pr

1)

Li
m

ite
d

Login ID:8FA03376/Student Name:ANSHULGARG/Overall Score:28

es

Explanation:
The opposite side in figure II is filled and the rest are empty. Hence,[2].

IM

Le

ar

ni

ng

es

ou

rc

Question: 17
In each of the following problems a related pair of figures is followed by 5 numbered pairs of figures. Select the pair that bears a
relationship dissimilar to that in the original pair. The best answer is to be selected from a group of fairly close choices.

1)

2)

3)

4)

5)

Explanation:

IMS Learning Resources Pvt.Ltd.,Mumbai.All copyrights to this material vestswith IMS Learning Resources Pvt.Ltd.
No part of this materials either in part oras a whole shall be copied,printed,electronically reproduced,sold or distributed without the written
consent of IMS Learing Resources Pvt.Ltd.and any such violation would entail initiation of suitable legal proceedings.

Copyright

ite
d

Login ID:8FA03376/Student Name:ANSHULGARG/Overall Score:28

3)

4)

5)

es

2)

ng

1)

ou

rc

es

Pr

iv
at
e

Li
m

Question: 18
In each of the following problems a related pair of figures is followed by 5 numbered pairs of figures. Select the pair that bears a
relationship similar to that in the original pair. The best answer is to be selected from a group of fairly close choices.

ar

ni

Explanation:
The number of objects in the 2ndpart is exactly double than the number of sides (straight lines) in the 1stpart. Hence, [5].

IM

Le

Question: 19
Find the figure from the figures numbered [1] to [5] that does not fit in series starting with figure marked as [X] and ending in figure marked
as [Y].

1)

2)

IMS Learning Resources Pvt.Ltd.,Mumbai.All copyrights to this material vestswith IMS Learning Resources Pvt.Ltd.
No part of this materials either in part oras a whole shall be copied,printed,electronically reproduced,sold or distributed without the written
consent of IMS Learing Resources Pvt.Ltd.and any such violation would entail initiation of suitable legal proceedings.

Copyright

Login ID:8FA03376/Student Name:ANSHULGARG/Overall Score:28

3)

4)

5)

Explanation:
Each circle moves 1, 2, 3, ... triangles ahead (clockwise) in consecutive figures. Hence, [4].

3)

4)

5)

es

2)

rc

ou

1)

Pr

iv
at
e

Li
m

ite
d

Question: 20
Choose the alternative that will replace the question mark (?) in the given series.

es

Explanation:
The shaded triangle is rotating to 2 places in clockwise direction in step 1, 3 places in step 2, 4 place in step 3 and 5 places in step 4 with
shaded circle located diagonally opposite this triangle. Hence, [2].

2)

1)

Le

ar

ni

ng

Question: 21
From among the five figures given below, choose the figure that does not fit into the group.

3)

4)

5)

IM

Explanation:
Out of the three bottom elements, the second element is the water image of the left, i.e., the first element. Also, the first element is rotated
45oanticlockwise and kept at the bottom right position. This is not seen in [3]. Hence, [3].

IMS Learning Resources Pvt.Ltd.,Mumbai.All copyrights to this material vestswith IMS Learning Resources Pvt.Ltd.
No part of this materials either in part oras a whole shall be copied,printed,electronically reproduced,sold or distributed without the written
consent of IMS Learing Resources Pvt.Ltd.and any such violation would entail initiation of suitable legal proceedings.

Copyright

Login ID:8FA03376/Student Name:ANSHULGARG/Overall Score:28

2)

3)

4)

5)

Pr

es

1)

iv
at
e

Li
m

ite
d

Question: 22
Each of the following questions consists of unmarked figures followed by five figures marked 1, 2, 3, 4 and 5. Select a figure from the
marked figures which will continue the series established by the unmarked figures.

es

ou

rc

Explanation:
The top left symbol in the 1stblock rotates 45oclockwise and also moves along the corners in a clockwise direction. '&' rotates 90o
clockwise and inverts laterally in alternate steps and also moves along the corners in a clockwise direction in each step. The square with
the shaded portion rotates 90oclcokwise as well as moves one step in the anticlockwise direction. Hence, [4].

IM

Le

ar

ni

ng

Question: 23
In each of the following problems, a related pair of figures is given followed by five numbered pairs of figures (1), (2), (3), (4) & (5). Select
a pair that has a relationship similar to that in the original pair.

1)

2)

3)

IMS Learning Resources Pvt.Ltd.,Mumbai.All copyrights to this material vestswith IMS Learning Resources Pvt.Ltd.
No part of this materials either in part oras a whole shall be copied,printed,electronically reproduced,sold or distributed without the written
consent of IMS Learing Resources Pvt.Ltd.and any such violation would entail initiation of suitable legal proceedings.

Copyright

Login ID:8FA03376/Student Name:ANSHULGARG/Overall Score:28

4)

5)

Explanation:
If the first figure is divided into 'n' parts, the second figure is an 'n1' sided polygon. Hence, [1].

2)

3)

4)

5)

iv
at
e

Pr

1)

Li
m

ite
d

Question: 24
Each question below has seven figures out of which five are marked 1 to 5. Choose the figure that does not fit in with the series.

ou

rc

es

Explanation:
In this series two steps are followed. The empty square moves one step in clockwise direction, a new symbol is added in the earlier empty
space and the remaining 2 symbols exchange their positions. In the 2ndstep the empty square moves diagonally, a new symbol is added in
the earlier empty space and the other 2 symbols exchange their positions. In [5] a new symbol is to be added at top left. Hence, [5].

IM

Le

ar

ni

ng

es

Question: 25
In each of the following problems, a related pair of figures is given followed by five numbered pairs of figures. Select a pair that has a
relationship similar to that in the original pair.

1)

2)

3)

4)

5)

IMS Learning Resources Pvt.Ltd.,Mumbai.All copyrights to this material vestswith IMS Learning Resources Pvt.Ltd.
No part of this materials either in part oras a whole shall be copied,printed,electronically reproduced,sold or distributed without the written
consent of IMS Learing Resources Pvt.Ltd.and any such violation would entail initiation of suitable legal proceedings.

Copyright

Login ID:8FA03376/Student Name:ANSHULGARG/Overall Score:28

Explanation:
The figure which is outside becomes the inner one. And figure which is inside becomes the outer one. Hence, [5].

2)

3)

4)

5)

iv
at
e

1)

Li
m

ite
d

Question: 26
Each question below has seven figures out of which five are marked 1 to 5. Choose the figure that does not fit in with the series.

es

Pr

Explanation:
The elements in the corners invert laterally and vertically both and rotate 90oanticlockwise in alternate steps while the remaining elements
invert laterally and rotate 90oclockwise in alternate steps. Hence, [5].

rc

Question: 27
In the following question, the word is coded according to a certain rule. Decode the rule and answer the question.
If APPLE is coded as JXWRJ, then what will be the code for NOKIA?
WWROG

2)

WWROF

3)

WXROF

4)

WWSOG

5)

None of these

ng

es

ou

1)

Le

ar

ni

Explanation:

IM

Question: 28
Four of the following five are alike in a certain way. Find the odd one which does not belong to the group.
1] 169
2] 289
3] 361
4] 531
5] 841

1)

IMS Learning Resources Pvt.Ltd.,Mumbai.All copyrights to this material vestswith IMS Learning Resources Pvt.Ltd.
No part of this materials either in part oras a whole shall be copied,printed,electronically reproduced,sold or distributed without the written
consent of IMS Learing Resources Pvt.Ltd.and any such violation would entail initiation of suitable legal proceedings.

Copyright

Login ID:8FA03376/Student Name:ANSHULGARG/Overall Score:28

2)

3)

4)

5)

Explanation:
All are squares of numbers i.e., 132= 169, 172= 289, 192= 361 and 292= 841 and except 531, which is not square of a number. Hence, [4].

3)

4)

5)

Pr

Explanation:
3200 = (80)2 2 = 1600 2,
1250 = (25)2 2 = 625 2.
All numbers are in (2n2) format except 2000. Hence, [2].

es

2)

rc

ou

1)

iv
at
e

Li
m

1] 3200
2] 2000
3] 1250
4] 1800
5] 2450

ite
d

Question: 29
Four of the following five are alike in a certain way. Find the odd one which does not belong to the group.

es

Question: 30
Four of the following five are alike in a certain way. Find the odd one which does not belong to the group.

2)

3)

4)

5)

Le

ar

1)

ni

ng

1] 1008
2] 1274
3] 2499
4] 3702
5] 4571

IM

Explanation:
All are divisible by 7 except 3702. Hence, [4].
Question: 31
Four of the following five are alike in a certain way. Find the odd one which does not belong to the group.
1] 216

IMS Learning Resources Pvt.Ltd.,Mumbai.All copyrights to this material vestswith IMS Learning Resources Pvt.Ltd.
No part of this materials either in part oras a whole shall be copied,printed,electronically reproduced,sold or distributed without the written
consent of IMS Learing Resources Pvt.Ltd.and any such violation would entail initiation of suitable legal proceedings.

Copyright

Login ID:8FA03376/Student Name:ANSHULGARG/Overall Score:28

1)

2)

3)

4)

5)

ite
d

2] 516
3] 1116
4] 1216
5] 4716

Li
m

Explanation:
1216 is not divisible by 3, others are. Hence, [4].

iv
at
e

Question: 32
Four of the following five are alike in a certain way. Find the odd one which does not belong to the group.

3)

4)

5)

es

2)

rc

es

Explanation:
All these are multiples of 303 except 9797. Hence, [4].

ou

1)

Pr

1] 606
2] 4545
3] 8181
4] 9797
5] 909

IM

Le

ar

ni

ng

Question: 33
Six sentences are given below. Rearrange them to form a coherent passage and answer the questions that follow.

IMS Learning Resources Pvt.Ltd.,Mumbai.All copyrights to this material vestswith IMS Learning Resources Pvt.Ltd.
No part of this materials either in part oras a whole shall be copied,printed,electronically reproduced,sold or distributed without the written
consent of IMS Learing Resources Pvt.Ltd.and any such violation would entail initiation of suitable legal proceedings.

Copyright

iv
at
e

Li
m

ite
d

Login ID:8FA03376/Student Name:ANSHULGARG/Overall Score:28

2)

3)

4)

5)

es

rc

1)

Pr

Which of these should be the first sentence after rearrangement?

ng

es

ou

Explanation:
F should be the opening sentence of the passage as it mentions the goals achieved by admitting new Asian observers to the Arctic
Council. Also, it is the only sentence which mentions the name of the council; all the other sentences refer to it as only the 'Council'. It
should be followed by C, which mentions the 'first' goal. 'This perspective' in A refers to the perspective introduced in C (it cannot be the
perspective in E as that is about considering another forum for discussions). A and D are about the disproportionate impact of the Arctic
region on the rest of the world, so they should come together. E and B are related to the second goal: E introduces it and B elaborates on
a point made in E (of the possibly more inclusive body where discussions can be transferred to). Thus, the correct sequence is FCADEB.
Hence, [5].

2)

3)

4)

Le

1)

ar

ni

Question: 34
Six sentences are given below. Rearrange them to form a coherent passage and answer the questions that follow.
Which of these should be the last sentence after rearrangement?

IM

5)

Explanation:
F should be the opening sentence of the passage as it mentions the goals achieved by admitting new Asian observers to the Arctic
Council. Also, it is the only sentence which mentions the name of the council; all the other sentences refer to it as only the 'Council'. It
should be followed by C, which mentions the 'first' goal. 'This perspective' in A refers to the perspective introduced in C (it cannot be the
perspective in E as that is about considering another forum for discussions). A and D are about the disproportionate impact of the Arctic
region on the rest of the world, so they should come together. E and B are related to the second goal: E introduces it and B elaborates on

IMS Learning Resources Pvt.Ltd.,Mumbai.All copyrights to this material vestswith IMS Learning Resources Pvt.Ltd.
No part of this materials either in part oras a whole shall be copied,printed,electronically reproduced,sold or distributed without the written
consent of IMS Learing Resources Pvt.Ltd.and any such violation would entail initiation of suitable legal proceedings.

Copyright

Login ID:8FA03376/Student Name:ANSHULGARG/Overall Score:28

a point made in E (of the possibly more inclusive body where discussions can be transferred to). Thus, the correct sequence is FCADEB.
Hence, [4].
Question: 35
Six sentences are given below. Rearrange them to form a coherent passage and answer the questions that follow.

2)

3)

4)

5)

Li
m

1)

ite
d

Which of these should precede D?

Pr

iv
at
e

Explanation:
F should be the opening sentence of the passage as it mentions the goals achieved by admitting new Asian observers to the Arctic
Council. Also, it is the only sentence which mentions the name of the council; all the other sentences refer to it as only the 'Council'. It
should be followed by C, which mentions the 'first' goal. 'This perspective' in A refers to the perspective introduced in C (it cannot be the
perspective in E as that is about considering another forum for discussions). A and D are about the disproportionate impact of the Arctic
region on the rest of the world, so they should come together. E and B are related to the second goal: E introduces it and B elaborates on
a point made in E (of the possibly more inclusive body where discussions can be transferred to). Thus, the correct sequence is FCADEB.
Hence, [1].

rc

es

Question: 36
Six sentences are given below. Rearrange them to form a coherent passage and answer the questions that follow.

3)

4)

5)

es

2)

ng

1)

ou

Which of these should be the penultimate sentence?

IM

Le

ar

ni

Explanation:
F should be the opening sentence of the passage as it mentions the goals achieved by admitting new Asian observers to the Arctic
Council. Also, it is the only sentence which mentions the name of the council; all the other sentences refer to it as only the 'Council'. It
should be followed by C, which mentions the 'first' goal. 'This perspective' in A refers to the perspective introduced in C (it cannot be the
perspective in E as that is about considering another forum for discussions). A and D are about the disproportionate impact of the Arctic
region on the rest of the world, so they should come together. E and B are related to the second goal: E introduces it and B elaborates on
a point made in E (of the possibly more inclusive body where discussions can be transferred to). Thus, the correct sequence is FCADEB.
Hence, [3].
Question: 37
Six sentences are given below. Rearrange them to form a coherent passage and answer the questions that follow.
Which of these should be the second sentence after rearrangement?
1)

2)

3)

IMS Learning Resources Pvt.Ltd.,Mumbai.All copyrights to this material vestswith IMS Learning Resources Pvt.Ltd.
No part of this materials either in part oras a whole shall be copied,printed,electronically reproduced,sold or distributed without the written
consent of IMS Learing Resources Pvt.Ltd.and any such violation would entail initiation of suitable legal proceedings.

Copyright

Login ID:8FA03376/Student Name:ANSHULGARG/Overall Score:28

4)

5)

ite
d

Explanation:
F should be the opening sentence of the passage as it mentions the goals achieved by admitting new Asian observers to the Arctic
Council. Also, it is the only sentence which mentions the name of the council; all the other sentences refer to it as only the 'Council'. It
should be followed by C, which mentions the 'first' goal. 'This perspective' in A refers to the perspective introduced in C (it cannot be the
perspective in E as that is about considering another forum for discussions). A and D are about the disproportionate impact of the Arctic
region on the rest of the world, so they should come together. E and B are related to the second goal: E introduces it and B elaborates on
a point made in E (of the possibly more inclusive body where discussions can be transferred to). Thus, the correct sequence is FCADEB.
Hence, [2].

3)

4)

5)

rc

2)

ou

es

1)

es

Pr

iv
at
e

Li
m

Question: 38
Each question is followed by two statements I and II. Answer each question using the following instructions.
Mark [1], if the question can be answered by using statement I alone.
Mark [2], if the question can be answered by using statement II alone.
Mark [3], if the question can be answered by using either of the statements alone.
Mark [4], if the question can be answered by using both the statements together, but cannot be answered by using either of the
statements alone.
Mark [5], if more data is required to answer the question.
On what date was Rajesh born?
I. He remembers that he was born between 4thJuly 1974 and 7thJuly 1974 (both dates excluded).
II. He remembers that he was born between 5thJuly 1974 and 8thJuly 1974 (both dates excluded).

ar

ni

ng

Explanation:
From statement I:
Rajesh was born either on 5th July or 6th July 1974. So statement I alone is not sufficient to answer the question.
From statement II:
Rajesh was born either on 6th July or 7th July 1974. So statement II alone is not sufficient to answer the question.
Now combining both statements, we get that Rajesh was born on 6th July 1974. So both statements are required to answer the question.
Hence, [4].

IM

Le

Question: 39
Each question is followed by two statements I and II. Answer each question using the following instructions.
Mark [1], if the question can be answered by using statement I alone.
Mark [2], if the question can be answered by using statement II alone.
Mark [3], if the question can be answered by using either of the statements alone.
Mark [4], if the question can be answered by using both the statements together, but cannot be answered by using either of the
statements alone.
Mark [5], if more data is required to answer the question.
In a linear arrangement, A, B, C, D, E, F and G occupy positions 1 to 7 (not necessarily in the same order). E occupies the middle
position. Who is 4 places away from B?
I.C and A are at extreme ends. D and F occupy the 3rdand 5thpositions (not necessarily in the same
order)

IMS Learning Resources Pvt.Ltd.,Mumbai.All copyrights to this material vestswith IMS Learning Resources Pvt.Ltd.
No part of this materials either in part oras a whole shall be copied,printed,electronically reproduced,sold or distributed without the written
consent of IMS Learing Resources Pvt.Ltd.and any such violation would entail initiation of suitable legal proceedings.

Copyright

Login ID:8FA03376/Student Name:ANSHULGARG/Overall Score:28

II.B is the only person in between F and A, who is at the extreme right end.
1)

2)

3)

4)

5)

es

Pr

iv
at
e

Li
m

ite
d

Explanation:

3)

4)

5)

IM

Explanation:

ar

2)

Le

1)

ni

ng

es

ou

rc

Question: 40
Each question is followed by two statements I and II. Answer each question using the following instructions.
Mark [1], if the question can be answered by using statement I alone.
Mark [2], if the question can be answered by using statement II alone.
Mark [3], if the question can be answered by using either of the statements alone.
Mark [4], if the question can be answered by using both the statements together, but cannot be answered by using either of the
statements alone.
Mark [5], if more data is required to answer the question.
In a family of members A, B, C, D, F and M, what is the relation between C and A?
I.F and M have two kids C and D. B is the spouse of C and A is the uncle of D.
II.C and B have one son and one daughter and they are A and D respectively. M and F are spouses of A and D respectively.

IMS Learning Resources Pvt.Ltd.,Mumbai.All copyrights to this material vestswith IMS Learning Resources Pvt.Ltd.
No part of this materials either in part oras a whole shall be copied,printed,electronically reproduced,sold or distributed without the written
consent of IMS Learing Resources Pvt.Ltd.and any such violation would entail initiation of suitable legal proceedings.

Copyright

Li
m

ite
d

Login ID:8FA03376/Student Name:ANSHULGARG/Overall Score:28

3)

4)

5)

es

2)

ng

1)

ou

rc

es

Pr

iv
at
e

Question: 41
Each question is followed by two statements I and II. Answer each question using the following instructions.
Mark [1], if the question can be answered by using statement I alone.
Mark [2], if the question can be answered by using statement II alone.
Mark [3], if the question can be answered by using either of the statements alone.
Mark [4], if the question can be answered by using both the statements together, but cannot be answered by using either of the
statements alone.
Mark [5], if more data is required to answer the question.
Ram and Laxman are the only two people travelling in a car. One of them owns the car. Who is the owner of the car?
I. The driver is not the owner of the car.
II. Ram is not driving the car.

Le

ar

ni

Explanation:
From statement I:
The owner is not the driver. But who is driving the car is not known. Statement I alone is not sufficient.
From statement II:
Ram is not driving which means Laxman is driving. But owner is not known. Statement II alone is not sufficient. Combining statements I
and II: Ram is the owner. Hence, [4].

IM

Question: 42
Refer to the data below and answer the questions that follow.

IMS Learning Resources Pvt.Ltd.,Mumbai.All copyrights to this material vestswith IMS Learning Resources Pvt.Ltd.
No part of this materials either in part oras a whole shall be copied,printed,electronically reproduced,sold or distributed without the written
consent of IMS Learing Resources Pvt.Ltd.and any such violation would entail initiation of suitable legal proceedings.

Copyright

iv
at
e

Li
m

ite
d

Login ID:8FA03376/Student Name:ANSHULGARG/Overall Score:28

3)

4)

5)

es

2)

rc

ou

1)

Pr

Arun's last year's annual income was 10.5 lakhs. He owns 2 cars. His wife's name is Sharda and they live in Chembur. He will turn 31 on
10.11.2010. Chembur is at a distance of 7 km from the club.

es

Explanation:
Arun satisfies all the conditions, except that he lives 7 km from the club. He isreferredto the Chairman of the Club. Hence, [4].

Question: 43
Refer to the data below and answer the questions that follow.

2)

3)

4)

5)

Le

ar

1)

ni

ng

Vineet applied for membership last year, but his application was rejected. He has recently bought a car called INDICA. The club is located
adjacent to the building where he stays in a rented flat. His wife, Ritu, lives in PUNE and they meet only on weekends. His previous year's
income is Rs. 12,75,000. His wife was born on 29.07.1981 and he was born exactly 9 months before her.

IM

Explanation:
It is mentioned that Vineet's application was rejected the previous year. Condition (iv) only specifies that he should live within 5 km of the
club and has no mention about living in a owned/ rented house. His age is more than 30 years (calculated from his wife's date of birth) and
the fact that his wife stays away from him during the week is no reason to reject his candidature. So he is selected. Hence, [1].
Question: 44
Refer to the data below and answer the questions that follow.

IMS Learning Resources Pvt.Ltd.,Mumbai.All copyrights to this material vestswith IMS Learning Resources Pvt.Ltd.
No part of this materials either in part oras a whole shall be copied,printed,electronically reproduced,sold or distributed without the written
consent of IMS Learing Resources Pvt.Ltd.and any such violation would entail initiation of suitable legal proceedings.

Copyright

Login ID:8FA03376/Student Name:ANSHULGARG/Overall Score:28

Aishwarya's last year's income was Rs. 25 lakhs. She lives 4.5 km away from the club and owns a MERCEDEZ car. Her date of birth is
29.06.1980 and she is engaged to Anand.
1)

2)

3)

4)

5)

ite
d

Explanation:
Aishwarya satisfies all the conditions except that she is not married. So she is referred to the Vice President of the Club. Hence, [3].

Li
m

Question: 45
Refer to the data below and answer the questions that follow.

2)

3)

4)

5)

Pr

es

1)

iv
at
e

Sumeet runs an advertising agency which, last year, had an annual turnover of Rs.2 crores. He has 2 cars, SKODA and ACCORD. He
lives in the upmarket area of BANDRA, which is 0.5km from the club and is married, with 2 kids. His age will be 31 years as on
01.04.2011.

ou

rc

Explanation:
Thestatement only mentions Sumeet's annual turnover and there is no mention about his annual income. Data provided is insufficient.
Hence, [5].

es

Question: 46
Refer to the data below and answer the questions that follow.

3)

4)

5)

ni

2)

ar

Le

1)

ng

Amar lives in a bungalow that is 2 km from the club. Amar lives out of the country for a great part of the year as he is travelling on work.
His previous years annual income was Rs.12 lakhs and he owns 3 premium bikes. He is just divorced from his wife.

IM

Explanation:
Amarowns 3 bikes and nocar, so his candidature is rejected. Hence, [2].
Question: 47
Read the short passages and answer the questions that follow.
A majority of the accidents that occur on the highway are caused by youngsters driving fast motorbikes.
What is the best course of action to be followed in the above situation?
1)

Motorbikes should be banned on the highways.

2)

Traffic police should be supplied with speedometers to check the speed of motorbikes.

IMS Learning Resources Pvt.Ltd.,Mumbai.All copyrights to this material vestswith IMS Learning Resources Pvt.Ltd.
No part of this materials either in part oras a whole shall be copied,printed,electronically reproduced,sold or distributed without the written
consent of IMS Learing Resources Pvt.Ltd.and any such violation would entail initiation of suitable legal proceedings.

Copyright

Login ID:8FA03376/Student Name:ANSHULGARG/Overall Score:28

3)

Heavy fines should be imposed on teenagers driving motorbikes.

4)

Motorbike driving licenses should not be issued to youngsters.

5)

None of the above

Explanation:
[1] and [4] are extreme and impractical. Simply checking the speed of motorbikes will not help prevent accidents. So, [2] is incomplete. [3]
is nonsensical as fines cannot be imposed for simply driving motorbikes. Hence, [5].

iv
at
e

Li
m

ite
d

Question: 48
Read the short passages and answer the questions that follow.
The war against tobacco has gained further vigour and momentum with the Ministry of Health and Family Welfare recently placing in the
public domain a draft Bill that seeks to amend the provisions of the Cigarettes and Other Tobacco Products (Prohibition of Advertisement
and Regulation of Trade and Commerce, Production, Supply and Distribution) Act, 2003. Among the changes proposed is the introduction
of the prohibition on using a 'name or brand of tobacco products for marketing or promoting goods and events'. It is one of the changes
that the government can easily implement and effectively enforce.
Which of the following has not been assumed by the drafters of the bill?
Established brands invoke positive associations in consumers irrespective of the products advertised.

2)

Established brands have a short term impact on the sales of the products advertised.

3)

Brands of tobacco products are conveniently used for advertising non-tobacco products.

4)

Brands are used to attract people to other non-tobacco products of the tobacco companies.

5)

None of the above

es

Pr

1)

es

ou

rc

Explanation:
As the bill is introducing the stated prohibition, the drafters would have assumed that established brands or names of tobacco products will
attract consumers even if those brands or names are used to promote other products and events. And as they do not want that to happen,
they are introducing the prohibition. [2] is not necessarily true: the impact is more likely long term. [3] and [4] would also be taken for
granted by the drafters. At best, they could be inferences drawn from this paragraph. Hence, [1].

Le

ar

ni

ng

Question: 49
Refer to the data below and answer the questions that follow.

IM

Who travelled at 90 kmph?


1)

2)

3)

4)

5)

Explanation:

IMS Learning Resources Pvt.Ltd.,Mumbai.All copyrights to this material vestswith IMS Learning Resources Pvt.Ltd.
No part of this materials either in part oras a whole shall be copied,printed,electronically reproduced,sold or distributed without the written
consent of IMS Learing Resources Pvt.Ltd.and any such violation would entail initiation of suitable legal proceedings.

Copyright

es

Pr

iv
at
e

Li
m

ite
d

Login ID:8FA03376/Student Name:ANSHULGARG/Overall Score:28

ou

Question: 50
Refer to the data below and answer the questions that follow.

3)

4)

5)

ng

2)

ni

es

Who travelled at 80 kmph?


1)

rc

Hence, [3]

IM

Le

ar

Explanation:

IMS Learning Resources Pvt.Ltd.,Mumbai.All copyrights to this material vestswith IMS Learning Resources Pvt.Ltd.
No part of this materials either in part oras a whole shall be copied,printed,electronically reproduced,sold or distributed without the written
consent of IMS Learing Resources Pvt.Ltd.and any such violation would entail initiation of suitable legal proceedings.

Copyright

es

Pr

iv
at
e

Li
m

ite
d

Login ID:8FA03376/Student Name:ANSHULGARG/Overall Score:28

820 km

3)

920 km

4)

860 km

5)

900 km

IM

Le

ar

Explanation:

2)

ng

880 km

ni

1)

es

What is the sum of the distances travelled by B and D?

ou

Question: 51
Refer to the data below and answer the questions that follow.

rc

Hence, [1]

IMS Learning Resources Pvt.Ltd.,Mumbai.All copyrights to this material vestswith IMS Learning Resources Pvt.Ltd.
No part of this materials either in part oras a whole shall be copied,printed,electronically reproduced,sold or distributed without the written
consent of IMS Learing Resources Pvt.Ltd.and any such violation would entail initiation of suitable legal proceedings.

Copyright

es

Pr

iv
at
e

Li
m

ite
d

Login ID:8FA03376/Student Name:ANSHULGARG/Overall Score:28

ou

Question: 52
Refer to the data below and answer the questions that follow.

rc

Hence, [3].

130 km

3)

140 km

4)

40 km

5)

20 km

IM

Le

ar

Explanation:

2)

ng

90 km

ni

1)

es

What is the difference between the distances travelled by A and C?

IMS Learning Resources Pvt.Ltd.,Mumbai.All copyrights to this material vestswith IMS Learning Resources Pvt.Ltd.
No part of this materials either in part oras a whole shall be copied,printed,electronically reproduced,sold or distributed without the written
consent of IMS Learing Resources Pvt.Ltd.and any such violation would entail initiation of suitable legal proceedings.

Copyright

es

Pr

iv
at
e

Li
m

ite
d

Login ID:8FA03376/Student Name:ANSHULGARG/Overall Score:28

1980 km

3)

2090 km

4)

2120 km

5)

2030 km

IM

Le

ar

Explanation:

2)

ng

1990 km

ni

1)

es

What is the total distance travelled by all of them together?

ou

Question: 53
Refer to the data below and answer the questions that follow.

rc

Hence, [2]

IMS Learning Resources Pvt.Ltd.,Mumbai.All copyrights to this material vestswith IMS Learning Resources Pvt.Ltd.
No part of this materials either in part oras a whole shall be copied,printed,electronically reproduced,sold or distributed without the written
consent of IMS Learing Resources Pvt.Ltd.and any such violation would entail initiation of suitable legal proceedings.

Copyright

rc

Total distance = 420 + 400 + 270 + 500 + 440 = 2030 km. Hence, [5].

es

Pr

iv
at
e

Li
m

ite
d

Login ID:8FA03376/Student Name:ANSHULGARG/Overall Score:28

es

ou

Question: 54
A statement that provides some information is followed by two other statements, numbered 1 and 2. Classify them on the basis of the
given options.
A stringent law passed last year has made motorcycle helmets compulsory in India and has had a huge impact on public health.
Statement 1: In Delhi alone, there are a total of 57,600 two-wheeler women riders.
Statement 2: The number of head injury cases has reduced by 50 percent over the past one year.
Statement 1 weakens the given information while statement 2 is neutral.

2)

Statement 1 is neutral while statement 2 strengthens the given information.

3)

Both statements are neutral.

4)

Both statements weaken the given information.

5)

Both statements strengthen the given information.

Le

ar

ni

ng

1)

IM

Explanation:
Statement 1 gives a statistic for the number of two-wheeler women riders in Delhi, which is unrelated to the context. Statement 2
highlights the positive impact on public health by citing the reduction in head injury incidents over the past year, when the law was
enforced, which suggests that the law is responsible for this change. Thus, statement 2 strengthens the argument. Hence, [2].
Question: 55
A statement that provides some information is followed by two other statements, numbered 1 and 2. Classify them on the basis of the
given options.
India's alarmingly high salt intake has contributed to a spike in hypertension and other cardiovascular diseases, mostly among its elderly
citizens.
Statement 1: Indians are innately predisposed towards a salty diet, which has accounted for the long and healthy life of the past
generation.

IMS Learning Resources Pvt.Ltd.,Mumbai.All copyrights to this material vestswith IMS Learning Resources Pvt.Ltd.
No part of this materials either in part oras a whole shall be copied,printed,electronically reproduced,sold or distributed without the written
consent of IMS Learing Resources Pvt.Ltd.and any such violation would entail initiation of suitable legal proceedings.

Copyright

Login ID:8FA03376/Student Name:ANSHULGARG/Overall Score:28

Statement 2: Most elderly citizens suffer from hypertension, which results from a combination of psychological, physical and genetic
factors.
1)

Statement 1 weakens the given information while statement 2 is neutral.

2)

Statement 1 is neutral while statement 2 strengthens the given information.

3)

Both statements are neutral.

4)

Both statements weaken the given information.

5)

Both statements strengthen the given information.

Li
m

ite
d

Explanation:
Statement 1 weakens the argument of high salt intake contributing to the increase in hypertension and heart diseases, by stating that high
salt-intake has been part of our diet and has contributed to the longevity of the past generation. Statement 2 gives completely different
reasons, viz. psychological, physical and genetic factors, for the spike in hypertension, which again weakens the main argument. Note that
though statement 2 doesn't address cardiovascular diseases, it still weakens part of the argument. Hence, [4].

Pr

iv
at
e

Question: 56
A statement that provides some information is followed by two other statements, numbered 1 and 2. Classify them on the basis of the
given options.
Selfies have become the greatest photographic trend of our time, enabling us to produce an image that comes closer to matching our
perception of what we think we look like.
Statement 1: In today's world, an enhanced self-image is what works in every sphere of life.
Statement 2: The current popularity of selfies will go a long way in boosting our self-esteem.
Statement 1 weakens the given information while statement 2 is neutral.

2)

Statement 1 is neutral while statement 2 strengthens the given information.

3)

Both statements are neutral.

4)

Both statements weaken the given information.

5)

Both statements strengthen the given information.

es

ou

rc

es

1)

ng

Explanation:
The argument is that selfies have become a significant trend that helps us reinforce a positive perception of ourselves. Statement 1 is a
general statement that correlates enhanced self-image with better prospects, but the perception of 'enhanced self-image' can arise from
other sources as well. So, it does not affect the argument. Statement 2 supports the argument by adding that selfies will lead to high selfesteem among people. Thus, it strengthens the argument. Hence, [2].

Le

ar

ni

Question: 57
Each question has some statements followed by a set of conclusions. Choose the conclusions that logically follow from the given
statements.

IM

Statements:
All young are smart. All smart are technicians.
Conclusions:
A. Some smart are not young.
B. Some technicians are not smart.
1)

Only A

2)

Only B

3)

Both A and B

4)

Either A or B

5)

Neither A nor B

IMS Learning Resources Pvt.Ltd.,Mumbai.All copyrights to this material vestswith IMS Learning Resources Pvt.Ltd.
No part of this materials either in part oras a whole shall be copied,printed,electronically reproduced,sold or distributed without the written
consent of IMS Learing Resources Pvt.Ltd.and any such violation would entail initiation of suitable legal proceedings.

Copyright

Login ID:8FA03376/Student Name:ANSHULGARG/Overall Score:28

iv
at
e

Li
m

ite
d

Explanation:

2)

Only B

3)

Both A and B

4)

Either A or B

5)

Neither A nor B

es

IM

Le

ar

ni

ng

Explanation:

es

Only A

1)

ou

rc

Statements:
All trains are metros. Some trains are monorails. No metro is cable.
Conclusions:
A. Some monorail is metro.
B. No metro is monorail.

Pr

Question: 58
Each question has some statements followed by a set of conclusions. Choose the conclusions that logically follow from the given
statements.

Question: 59
Each question has some statements followed by a set of conclusions. Choose the conclusions that logically follow from the given
statements.

IMS Learning Resources Pvt.Ltd.,Mumbai.All copyrights to this material vestswith IMS Learning Resources Pvt.Ltd.
No part of this materials either in part oras a whole shall be copied,printed,electronically reproduced,sold or distributed without the written
consent of IMS Learing Resources Pvt.Ltd.and any such violation would entail initiation of suitable legal proceedings.

Copyright

Login ID:8FA03376/Student Name:ANSHULGARG/Overall Score:28

Only A

2)

Only B

3)

Both A and B

4)

Either A or B

5)

Neither A nor B

Li
m

1)

ite
d

Statements:
Some students are teachers. Some teachers are principals. Some principals are lecturers.
Conclusions:
A. Some students are principals.
B. No students are principals.

es

ou

rc

es

Pr

iv
at
e

Explanation:

ni

ng

Question: 60
Each question has some statements followed by a set of conclusions. Choose the conclusions that logically follow from the given
statements.

Le

ar

Statements:
Some lamps are bulbs. All bulbs are CFLs. No CFLs are LEDs.
Conclusions:
A. No bulbs are LEDs.
B. Some lamps may be LEDs.

Only B

IM

2)

Only A

1)

3)

Both A and B

4)

Either A or B

5)

Neither A nor B

Explanation:

IMS Learning Resources Pvt.Ltd.,Mumbai.All copyrights to this material vestswith IMS Learning Resources Pvt.Ltd.
No part of this materials either in part oras a whole shall be copied,printed,electronically reproduced,sold or distributed without the written
consent of IMS Learing Resources Pvt.Ltd.and any such violation would entail initiation of suitable legal proceedings.

Copyright

Li
m

ite
d

Login ID:8FA03376/Student Name:ANSHULGARG/Overall Score:28

iv
at
e

Question: 61
Each question has some statements followed by a set of conclusions. Choose the conclusions that logically follow from the given
statements.

Only A

2)

Only B

3)

Both A and B

4)

Either A or B

5)

Neither A nor B

es

ou

rc

es

1)

Pr

Statements:
All trains are metros. Some trains are monorails. No metro is cable.
Conclusions:
A. Some monorail is cable.
B. No train is cable.

IM

Le

ar

ni

ng

Explanation:

Question: 62
Each question has some statements followed by a set of conclusions. Choose the conclusions that logically follow from the given
statements.
Statements:
Some students are teachers. Some teachers are principals. Some principals are lecturers.

IMS Learning Resources Pvt.Ltd.,Mumbai.All copyrights to this material vestswith IMS Learning Resources Pvt.Ltd.
No part of this materials either in part oras a whole shall be copied,printed,electronically reproduced,sold or distributed without the written
consent of IMS Learing Resources Pvt.Ltd.and any such violation would entail initiation of suitable legal proceedings.

Copyright

Login ID:8FA03376/Student Name:ANSHULGARG/Overall Score:28

1)

Only A

2)

Only B

3)

Both A and B

4)

Either A or B

5)

Neither A nor B

ite
d

Conclusions:
A. Some teachers may be lecturers.
B. No students are lecturers.

es

ou

rc

es

Pr

iv
at
e

Li
m

Explanation:

ng

Question: 63
Each question has some statements followed by a set of conclusions. Choose the conclusions that logically follow from the given
statements.

ar

ni

Statements:
Some lamps are bulbs. All bulbs are CFLs. No CFLs are LEDs.
Conclusions:
A. All lamps are CFLs.
B. Some CFLs are not bulbs.
Only A

2)

Only B

3)

Both A and B

IM

Le

1)

4)

Either A or B

5)

Neither A nor B

Explanation:

IMS Learning Resources Pvt.Ltd.,Mumbai.All copyrights to this material vestswith IMS Learning Resources Pvt.Ltd.
No part of this materials either in part oras a whole shall be copied,printed,electronically reproduced,sold or distributed without the written
consent of IMS Learing Resources Pvt.Ltd.and any such violation would entail initiation of suitable legal proceedings.

Copyright

ite
d

Login ID:8FA03376/Student Name:ANSHULGARG/Overall Score:28

ng

es

ou

rc

es

Pr

iv
at
e

Li
m

Question: 64
Read the following information followed by six statements carefully and answer the questions that follow.

3)

4)

5)

ar

2)

Le

1)

ni

What is the assumption of the school officials?

IM

Explanation:
The school officials must assume that the physical education program will lead to changes in the lifestyle and nutrition of the students; that
is why they are introducing it. All the remaining options, though they might be true, need not have been necessarily assumed by the
officials. Hence, [1].
Question: 65
Read the following information followed by six statements carefully and answer the questions that follow.
Which of the following will adversely impact the plan?

IMS Learning Resources Pvt.Ltd.,Mumbai.All copyrights to this material vestswith IMS Learning Resources Pvt.Ltd.
No part of this materials either in part oras a whole shall be copied,printed,electronically reproduced,sold or distributed without the written
consent of IMS Learing Resources Pvt.Ltd.and any such violation would entail initiation of suitable legal proceedings.

Copyright

Login ID:8FA03376/Student Name:ANSHULGARG/Overall Score:28

1)

2)

3)

4)

5)

None of the above

ite
d

Explanation:
None of the options will adversely impact the plan of requiring students to take part in mandatory and varied physical education programs.
Hence, [5].

Pr

iv
at
e

Li
m

Question: 66
Refer to the data below and answer the questions that follow.
There are 6 bags, numbered 1, 2, 3, 4, 5 and 6, arranged in serial order, from left to right, on a table. These bags contain 1 ball each from
the 6 balls Red, Blue, Green, Orange, Yellow and Violet not necessarily in the same order.
There are the following constraints:
i. The Green Ball is not in Bag 2.
ii. The Orange Ball is either in Bag 1 or Bag 6.
iii. The Green Ball and the Violet Ball are not in adjacent bags.
iv. The Yellow Ball is in a bag that is immediately left of the bag that contains the Red Ball.

2)

Yellow, Red, Violet, Blue, Green, Orange

3)

Orange, Green, Yellow, Red, Violet, Blue

4)

Orange, Red, Violet, Green, Blue, Yellow

5)

Orange, Violet, Blue, Yellow, Green, Red

ng

es

Explanation:
Option [1] does not satisfy Constraint (iii).
Option [3] does not satisfy Constraint (i).
Option [4] and Option [5] do not satisfy Constraint (iv).
Only Option [2] satisfies all the constraints. Hence, [2].

rc

Yellow, Red, Green, Violet, Blue, Orange

ou

1)

es

Which of the following could be the order of the bags from 1 to 6?

ar

ni

Question: 67
Refer to the data below and answer the questions that follow.

1)

2)

3)

Le

If the Violet Ball is in Bag 4, then the Blue Ball is in which bag?

IM

4)

5)

Explanation:

IMS Learning Resources Pvt.Ltd.,Mumbai.All copyrights to this material vestswith IMS Learning Resources Pvt.Ltd.
No part of this materials either in part oras a whole shall be copied,printed,electronically reproduced,sold or distributed without the written
consent of IMS Learing Resources Pvt.Ltd.and any such violation would entail initiation of suitable legal proceedings.

Copyright

Login ID:8FA03376/Student Name:ANSHULGARG/Overall Score:28

ite
d

Question: 68
Refer to the data below and answer the questions that follow.

1)

Orange

2)

Green

3)

Violet

4)

Yellow

5)

None of these

iv
at
e

Li
m

Which of the following balls can not be kept in Bag 6?

Pr

Explanation:
The Yellow Ball is always to the left of the red Ball; so, the Yellow Ball cannot be in Bag 6. Hence, [4].

3)

4)

5)

rc

es

2)

ng

1)

ou

If the Blue Ball is in Bag 3, then the Violet Ball is in which bag?

es

Question: 69
Refer to the data below and answer the questions that follow.

IM

Le

ar

ni

Explanation:

Question: 70
Refer to the data below and answer the questions that follow.
If the Green Ball is in Bag 4, balls of which of the following colours are always in bags adjacent to each other?
1)

Yellow and Violet

IMS Learning Resources Pvt.Ltd.,Mumbai.All copyrights to this material vestswith IMS Learning Resources Pvt.Ltd.
No part of this materials either in part oras a whole shall be copied,printed,electronically reproduced,sold or distributed without the written
consent of IMS Learing Resources Pvt.Ltd.and any such violation would entail initiation of suitable legal proceedings.

Copyright

Login ID:8FA03376/Student Name:ANSHULGARG/Overall Score:28

2)

Green and Blue

3)

Red and Green

4)

Orange and Green

5)

None of these

Explanation:
When the Green Ball is in Bag 4, the only possible combinations are VYRGBO, OYRGBV and OVBGYR. Hence, [2].

3)

4)

5)

es

2)

rc

ou

1)

Pr

iv
at
e

Li
m

ite
d

Question: 71
Four out of five sentences contribute to one main idea. Choose the one that is not related to the others.

es

Explanation:
All sentences except [3] talk about how reading good writing helps one become a better writer. [3] talks about why anyone (not just
writers) reads. Hence, [3].

IM

Le

ar

ni

ng

Question: 72
Four out of five sentences contribute to one main idea. Choose the one that is not related to the others.

1)

IMS Learning Resources Pvt.Ltd.,Mumbai.All copyrights to this material vestswith IMS Learning Resources Pvt.Ltd.
No part of this materials either in part oras a whole shall be copied,printed,electronically reproduced,sold or distributed without the written
consent of IMS Learing Resources Pvt.Ltd.and any such violation would entail initiation of suitable legal proceedings.

Copyright

Login ID:8FA03376/Student Name:ANSHULGARG/Overall Score:28

2)

3)

4)

5)

ite
d

Explanation:
All sentences except [3] are related to India's rail plans and China's involvement in the same. [3], which talks about a Chinese company
losing a contract, digresses from the main idea. Hence, [3].

3)

4)

5)

es

2)

rc

ou

1)

Pr

iv
at
e

Li
m

Question: 73
Four out of five sentences contribute to one main idea. Choose the one that is not related to the others.

es

Explanation:
[5] talks about the importance of art for humans. [4], [1] and [3] elaborate on the benefits of art on children. [2], which talks about people in
creative roles being role models, is the one that distracts from the main idea. Hence, [2].

IM

Le

ar

ni

ng

Question: 74
Four out of five sentences contribute to one main idea. Choose the one that is not related to the others.

1)

2)

3)

4)

5)

Explanation:

IMS Learning Resources Pvt.Ltd.,Mumbai.All copyrights to this material vestswith IMS Learning Resources Pvt.Ltd.
No part of this materials either in part oras a whole shall be copied,printed,electronically reproduced,sold or distributed without the written
consent of IMS Learing Resources Pvt.Ltd.and any such violation would entail initiation of suitable legal proceedings.

Copyright

Login ID:8FA03376/Student Name:ANSHULGARG/Overall Score:28

All sentences except [4] discuss observable and measurable behaviours. [4] talks about guessing the reasons behind someone's
behaviour, without sufficient evidence. Hence, [4].

2)

3)

4)

5)

Pr

es

1)

iv
at
e

Li
m

ite
d

Question: 75
Four out of five sentences contribute to one main idea. Choose the one that is not related to the others.

ou

rc

Explanation:
Only [2] talks about curriculum development research; all the other sentences are about curriculums in general. Hence, [2].

ar

ni

ng

es

Question: 76
Refer to the data below and answer the questions that follow.
Five men, viz. M1, M2, M3, M4 and M5, and four women, viz. W1, W2, W3 and W4, are sitting in a row, facing the same direction and all
equidistant from each other. Further information is known about them.
1.M2 sits exactly between M4 and M5.
2.W2 sits second to the right of W3.
3. M3, who does not sit at any of the extreme ends, has no woman seatedto his left.
4.W2 sits at one of the extreme ends.
5.M1 has fewer women seated to his right than to his left.
6.W1 has equal number of people to her right and to herleft.

1)

2)

3)

Le

How many people sit to the right of W4?

IM

4)

5)

Cannot be determined

Explanation:

IMS Learning Resources Pvt.Ltd.,Mumbai.All copyrights to this material vestswith IMS Learning Resources Pvt.Ltd.
No part of this materials either in part oras a whole shall be copied,printed,electronically reproduced,sold or distributed without the written
consent of IMS Learing Resources Pvt.Ltd.and any such violation would entail initiation of suitable legal proceedings.

Copyright

Login ID:8FA03376/Student Name:ANSHULGARG/Overall Score:28

3. Hence, [3].

ite
d

Question: 77
Refer to the data below and answer the questions that follow.
Who sits three places to the right of M5?
M3

2)

M5

3)

M4

4)

M2

5)

Cannot be determined

iv
at
e

Li
m

1)

rc

es

Pr

Explanation:

ou

Cannot be determined. Hence, [5]

es

Question: 78
Refer to the data below and answer the questions that follow.

Which of the following pairs can sit adjacent to each other?


M3M4

2)

M1M3

3)

M1M2

4)

W4M5

5)

None of these

Le

ar

ni

ng

1)

IM

Explanation:

M3 and M4 can sit adjacent to each other. Hence, [1].


Question: 79
Refer to the data below and answer the questions that follow.

IMS Learning Resources Pvt.Ltd.,Mumbai.All copyrights to this material vestswith IMS Learning Resources Pvt.Ltd.
No part of this materials either in part oras a whole shall be copied,printed,electronically reproduced,sold or distributed without the written
consent of IMS Learing Resources Pvt.Ltd.and any such violation would entail initiation of suitable legal proceedings.

Copyright

Login ID:8FA03376/Student Name:ANSHULGARG/Overall Score:28

Where does W3 sit?


1)

Second to the left of W1

2)

Immediate left of M1

3)

Immediate right of M3

4)

Third to the right of W1

5)

None of these

Li
m

ite
d

Explanation:

iv
at
e

Immediate left of M1. Hence, [2].


Question: 80
Refer to the data below and answer the questions that follow.

M4

2)

M5

3)

W1

4)

W4

5)

Cannot be determined

ou

rc

es

1)

Pr

Who among the following definitely sits next to M3?

ng

es

Explanation:

ar

ni

W1. Hence, [3].

Le

Question: 81
Refer to the data below and answer the questions that follow.
Of the given nine people, the exact positions of how many people can be correctly determined?
5

2)

1)

3)

4)

5)

IM

Explanation:

IMS Learning Resources Pvt.Ltd.,Mumbai.All copyrights to this material vestswith IMS Learning Resources Pvt.Ltd.
No part of this materials either in part oras a whole shall be copied,printed,electronically reproduced,sold or distributed without the written
consent of IMS Learing Resources Pvt.Ltd.and any such violation would entail initiation of suitable legal proceedings.

Copyright

Login ID:8FA03376/Student Name:ANSHULGARG/Overall Score:28

7. Hence, [3].

ite
d

Question: 82
Refer to the data below and answer the questions that follow.
Who sits to the immediate left of M2?
M3

2)

M4

3)

M5

4)

W4

5)

Cannot be determined

iv
at
e

Li
m

1)

rc

es

Pr

Explanation:

ou

Cannot be determined. Hence, [5].

Le

ar

ni

ng

es

Question: 83
Refer to the data below and answer the questions that follow.
A circular-shaped necklace with 12 pendants numbered 1 to 12 arranged around it, is placed flat on a table. The distance between any 2
adjacent pendants is the same. Following information is also available.
1. Moving clockwise from pendant 12 to pendant 9, there are 3 pendants (neither of which is pendant 11 nor pendant 6). Also, the number
on these 3 pendants are increasing in value as we move in the clockwise direction.
2. Pendant 4 is to the immediate right of pendant 1 and 3 places to the left of pendant 3.
3. Pendant 5 and 8 are adjacent to each other.
4. Pendant 2 is opposite to pendant 10.
5. Moving clockwise from pendant 11 to pendant 7, there are only 2 pendants, one of which is pendant 10.

1)

Pendant 6

2)

Which pendant is between pendant 11 and pendant 7 if we move clockwise from pendant 11?

IM

Pendant 4

3)

Pendant 3

4)

Pendant 8

5)

Pendant 1

Explanation:

IMS Learning Resources Pvt.Ltd.,Mumbai.All copyrights to this material vestswith IMS Learning Resources Pvt.Ltd.
No part of this materials either in part oras a whole shall be copied,printed,electronically reproduced,sold or distributed without the written
consent of IMS Learing Resources Pvt.Ltd.and any such violation would entail initiation of suitable legal proceedings.

Copyright

IM

Le

ar

ni

ng

es

ou

rc

es

Pr

iv
at
e

Li
m

ite
d

Login ID:8FA03376/Student Name:ANSHULGARG/Overall Score:28

Pendant 3. Hence, [3].


Question: 84
Refer to the data below and answer the questions that follow.

IMS Learning Resources Pvt.Ltd.,Mumbai.All copyrights to this material vestswith IMS Learning Resources Pvt.Ltd.
No part of this materials either in part oras a whole shall be copied,printed,electronically reproduced,sold or distributed without the written
consent of IMS Learing Resources Pvt.Ltd.and any such violation would entail initiation of suitable legal proceedings.

Copyright

Login ID:8FA03376/Student Name:ANSHULGARG/Overall Score:28

Which pendant is to the immediate right of pendant 12?


1)

Pendant 6

2)

Pendant 2

3)

Pendant 5

4)

Pendant 1

5)

None of these

IM

Le

ar

ni

ng

es

ou

rc

es

Pr

iv
at
e

Li
m

ite
d

Explanation:

IMS Learning Resources Pvt.Ltd.,Mumbai.All copyrights to this material vestswith IMS Learning Resources Pvt.Ltd.
No part of this materials either in part oras a whole shall be copied,printed,electronically reproduced,sold or distributed without the written
consent of IMS Learing Resources Pvt.Ltd.and any such violation would entail initiation of suitable legal proceedings.

Copyright

IM

Le

ar

ni

ng

es

ou

rc

es

Pr

iv
at
e

Li
m

ite
d

Login ID:8FA03376/Student Name:ANSHULGARG/Overall Score:28

Pendant 6. Hence, [1].


Question: 85
Refer to the data below and answer the questions that follow.

IMS Learning Resources Pvt.Ltd.,Mumbai.All copyrights to this material vestswith IMS Learning Resources Pvt.Ltd.
No part of this materials either in part oras a whole shall be copied,printed,electronically reproduced,sold or distributed without the written
consent of IMS Learing Resources Pvt.Ltd.and any such violation would entail initiation of suitable legal proceedings.

Copyright

Login ID:8FA03376/Student Name:ANSHULGARG/Overall Score:28

Which pendant is opposite pendant 8?


1)

Pendant 1

2)

Pendant 4

3)

Pendant 7

4)

Pendant 6

5)

None of these

IM

Le

ar

ni

ng

es

ou

rc

es

Pr

iv
at
e

Li
m

ite
d

Explanation:

IMS Learning Resources Pvt.Ltd.,Mumbai.All copyrights to this material vestswith IMS Learning Resources Pvt.Ltd.
No part of this materials either in part oras a whole shall be copied,printed,electronically reproduced,sold or distributed without the written
consent of IMS Learing Resources Pvt.Ltd.and any such violation would entail initiation of suitable legal proceedings.

Copyright

IM

Le

ar

ni

ng

es

ou

rc

es

Pr

iv
at
e

Li
m

ite
d

Login ID:8FA03376/Student Name:ANSHULGARG/Overall Score:28

Pendant 4. Hence, [2].


Question: 86
Refer to the data below and answer the questions that follow.

IMS Learning Resources Pvt.Ltd.,Mumbai.All copyrights to this material vestswith IMS Learning Resources Pvt.Ltd.
No part of this materials either in part oras a whole shall be copied,printed,electronically reproduced,sold or distributed without the written
consent of IMS Learing Resources Pvt.Ltd.and any such violation would entail initiation of suitable legal proceedings.

Copyright

Login ID:8FA03376/Student Name:ANSHULGARG/Overall Score:28

Which of the following statements is true?


1)

Moving clockwise from pendant 2, pendant 5 is the only pendant between pendant 2 and pendant 9

2)

Moving anti-clockwise from pendant 6, pendant 4 is the only pendant between pendant 6 and pendant 7

3)

Moving clockwise from pendant 1, pendant 6 is the only pendant between pendant 1 and pendant 12

4)

Moving anti-clockwise from pendant 8, pendant 5 is the only pendant between pendant 8 and pendant 12

5)

None of these

IM

Le

ar

ni

ng

es

ou

rc

es

Pr

iv
at
e

Li
m

ite
d

Explanation:

IMS Learning Resources Pvt.Ltd.,Mumbai.All copyrights to this material vestswith IMS Learning Resources Pvt.Ltd.
No part of this materials either in part oras a whole shall be copied,printed,electronically reproduced,sold or distributed without the written
consent of IMS Learing Resources Pvt.Ltd.and any such violation would entail initiation of suitable legal proceedings.

Copyright

IM

Le

ar

ni

ng

es

ou

rc

es

Pr

iv
at
e

Li
m

ite
d

Login ID:8FA03376/Student Name:ANSHULGARG/Overall Score:28

Statement [3] is true. Hence, [3].


Question: 87
Refer to the data below and answer the questions that follow.

IMS Learning Resources Pvt.Ltd.,Mumbai.All copyrights to this material vestswith IMS Learning Resources Pvt.Ltd.
No part of this materials either in part oras a whole shall be copied,printed,electronically reproduced,sold or distributed without the written
consent of IMS Learing Resources Pvt.Ltd.and any such violation would entail initiation of suitable legal proceedings.

Copyright

Login ID:8FA03376/Student Name:ANSHULGARG/Overall Score:28

Which pair of pendants are not adjacent to each other?


1)

1 and 6

2)

5 and 8

3)

9 and 11

4)

12 and 2

5)

None of these

IM

Le

ar

ni

ng

es

ou

rc

es

Pr

iv
at
e

Li
m

ite
d

Explanation:

IMS Learning Resources Pvt.Ltd.,Mumbai.All copyrights to this material vestswith IMS Learning Resources Pvt.Ltd.
No part of this materials either in part oras a whole shall be copied,printed,electronically reproduced,sold or distributed without the written
consent of IMS Learing Resources Pvt.Ltd.and any such violation would entail initiation of suitable legal proceedings.

Copyright

IM

Le

ar

ni

ng

es

ou

rc

es

Pr

iv
at
e

Li
m

ite
d

Login ID:8FA03376/Student Name:ANSHULGARG/Overall Score:28

All the given pairs are adjacent to each other. Hence, [5].
Question: 88
Refer to the data below and answer the questions that follow.

IMS Learning Resources Pvt.Ltd.,Mumbai.All copyrights to this material vestswith IMS Learning Resources Pvt.Ltd.
No part of this materials either in part oras a whole shall be copied,printed,electronically reproduced,sold or distributed without the written
consent of IMS Learing Resources Pvt.Ltd.and any such violation would entail initiation of suitable legal proceedings.

Copyright

Login ID:8FA03376/Student Name:ANSHULGARG/Overall Score:28

Moving clockwise, the pendant/s between Pendant 7 and Pendant 6 is/are:


1)

Only 1

2)

Only 4

3)

Both 1 and 4

4)

Both 1 and 3

5)

None of these

IM

Le

ar

ni

ng

es

ou

rc

es

Pr

iv
at
e

Li
m

ite
d

Explanation:

IMS Learning Resources Pvt.Ltd.,Mumbai.All copyrights to this material vestswith IMS Learning Resources Pvt.Ltd.
No part of this materials either in part oras a whole shall be copied,printed,electronically reproduced,sold or distributed without the written
consent of IMS Learing Resources Pvt.Ltd.and any such violation would entail initiation of suitable legal proceedings.

Copyright

IM

Le

ar

ni

ng

es

ou

rc

es

Pr

iv
at
e

Li
m

ite
d

Login ID:8FA03376/Student Name:ANSHULGARG/Overall Score:28

Both 1 and 4. Hence, [3].


Question: 89
Refer to the data below and answer the questions that follow.

IMS Learning Resources Pvt.Ltd.,Mumbai.All copyrights to this material vestswith IMS Learning Resources Pvt.Ltd.
No part of this materials either in part oras a whole shall be copied,printed,electronically reproduced,sold or distributed without the written
consent of IMS Learing Resources Pvt.Ltd.and any such violation would entail initiation of suitable legal proceedings.

Copyright

Login ID:8FA03376/Student Name:ANSHULGARG/Overall Score:28

If the positions of pendant 5 and pendant 7 are interchanged, then pendant 5 will be:
1)

Opposite pendant 12

2)

Opposite pendant 8

3)

To the immediate right of pendant 6

4)

To the immediate left of pendant 10

5)

None of these

IM

Le

ar

ni

ng

es

ou

rc

es

Pr

iv
at
e

Li
m

ite
d

Explanation:

IMS Learning Resources Pvt.Ltd.,Mumbai.All copyrights to this material vestswith IMS Learning Resources Pvt.Ltd.
No part of this materials either in part oras a whole shall be copied,printed,electronically reproduced,sold or distributed without the written
consent of IMS Learing Resources Pvt.Ltd.and any such violation would entail initiation of suitable legal proceedings.

Copyright

IM

Le

ar

ni

ng

es

ou

rc

es

Pr

iv
at
e

Li
m

ite
d

Login ID:8FA03376/Student Name:ANSHULGARG/Overall Score:28

Pendant 5 will be to the immediate left of pendant 10. Hence, [4].


Question: 90
Refer to the data below and answer the questions that follow.

IMS Learning Resources Pvt.Ltd.,Mumbai.All copyrights to this material vestswith IMS Learning Resources Pvt.Ltd.
No part of this materials either in part oras a whole shall be copied,printed,electronically reproduced,sold or distributed without the written
consent of IMS Learing Resources Pvt.Ltd.and any such violation would entail initiation of suitable legal proceedings.

Copyright

Login ID:8FA03376/Student Name:ANSHULGARG/Overall Score:28

Li
m

ite
d

In a family of 12 that spans across three generations, there are 4 married couples and 4 unmarried individuals. The following information
is known.
K and L are uncle and aunt respectively of M, who is a female.
J is the maternal grandfather of U and V, who are twin brothers.
D is the mother-in-law of S.
R and M are female cousins.
All married couples have children.
None of the individuals in the third generation are married.
J has two sons and a daughter.
S, who is married into the family, is the brother-in-law of B.
P is the sister-in-law of L, who has two children.
R is the only niece of T.

2)

3)

4)

5)

None of these

Pr

1)

iv
at
e

How many females are there in the family?

Le

ar

ni

ng

es

ou

rc

es

Explanation:

Hence, [3]

IM

Question: 91
Refer to the data below and answer the questions that follow.
How is K related to V?
1)

Uncle

2)

Aunt

3)

Brother-in-law

4)

Nephew

IMS Learning Resources Pvt.Ltd.,Mumbai.All copyrights to this material vestswith IMS Learning Resources Pvt.Ltd.
No part of this materials either in part oras a whole shall be copied,printed,electronically reproduced,sold or distributed without the written
consent of IMS Learing Resources Pvt.Ltd.and any such violation would entail initiation of suitable legal proceedings.

Copyright

Login ID:8FA03376/Student Name:ANSHULGARG/Overall Score:28

5)

None of these

iv
at
e

Li
m

ite
d

Explanation:

Pr

Hence, [1]

es

Question: 92
Refer to the data below and answer the questions that follow.

rc

How is L related to D?
Mother

2)

Daughter

3)

Sister-in-law

4)

Niece

5)

None of these

es

ou

1)

IM

Le

ar

ni

ng

Explanation:

Hence, [2]

IMS Learning Resources Pvt.Ltd.,Mumbai.All copyrights to this material vestswith IMS Learning Resources Pvt.Ltd.
No part of this materials either in part oras a whole shall be copied,printed,electronically reproduced,sold or distributed without the written
consent of IMS Learing Resources Pvt.Ltd.and any such violation would entail initiation of suitable legal proceedings.

Copyright

Login ID:8FA03376/Student Name:ANSHULGARG/Overall Score:28

Question: 93
Refer to the data below and answer the questions that follow.

1)

Brother

2)

Sister-in-law

3)

Husband

4)

Brother-in-law

5)

None of these

ite
d

How is B related to P?

es

ou

rc

es

Pr

iv
at
e

Li
m

Explanation:

Hence, [4]

ng

Question: 94
Refer to the data below and answer the questions that follow.

3)

4)

5)

IM

Explanation:

ar

2)

Le

1)

ni

Who among the following has twin children?

IMS Learning Resources Pvt.Ltd.,Mumbai.All copyrights to this material vestswith IMS Learning Resources Pvt.Ltd.
No part of this materials either in part oras a whole shall be copied,printed,electronically reproduced,sold or distributed without the written
consent of IMS Learing Resources Pvt.Ltd.and any such violation would entail initiation of suitable legal proceedings.

Copyright

iv
at
e

Li
m

ite
d

Login ID:8FA03376/Student Name:ANSHULGARG/Overall Score:28

Hence, [5]

Pr

Question: 95
Refer to the data below and answer the questions that follow.

LT

3)

TS

4)

PK

5)

KD

rc

2)

ou

LK

es

1)

es

Which of the following are married couples?

IM

Le

ar

ni

ng

Explanation:

Hence, [4].
Question: 96

IMS Learning Resources Pvt.Ltd.,Mumbai.All copyrights to this material vestswith IMS Learning Resources Pvt.Ltd.
No part of this materials either in part oras a whole shall be copied,printed,electronically reproduced,sold or distributed without the written
consent of IMS Learing Resources Pvt.Ltd.and any such violation would entail initiation of suitable legal proceedings.

Copyright

Login ID:8FA03376/Student Name:ANSHULGARG/Overall Score:28

In the following questions, the words are coded according to a certain rule. Decode the rule and answer each question.

1)

ECQKJY

2)

ECQKIY

3)

ECQKIX

4)

ECQKJX

5)

None of these

ite
d

If MAHESH is coded as NCKIXN, then what will be the code for DANGER?

iv
at
e

Li
m

Explanation:

If PAJERO is coded as NCHGPQ, then what will be the code for SURESH?
QWQGQJ

2)

QWPGSJ

3)

QWPGQJ

4)

QXPGQJ

5)

None of these

ou

rc

es

1)

Pr

Question: 97
In the following questions, the words are coded according to a certain rule. Decode the rule and answer each question.

ni

ng

es

Explanation:

Le

ar

Question: 98
In the following questions, the words are coded according to a certain rule. Decode the rule and answer each question.

1)

BRITAIN

2)

If ENGLAND is coded as 5573154, then what can be coded as 3961291?

IM

FINLAND

3)

CROATIA

4)

AMERICA

5)

None of these

Explanation:
ENGLAND is coded as 5573154. Close observation reveals that the each letter is coded by the sum of the digits in the numerical value.
Thus, CROATIA can be coded as 3961291. Hence, [3].

IMS Learning Resources Pvt.Ltd.,Mumbai.All copyrights to this material vestswith IMS Learning Resources Pvt.Ltd.
No part of this materials either in part oras a whole shall be copied,printed,electronically reproduced,sold or distributed without the written
consent of IMS Learing Resources Pvt.Ltd.and any such violation would entail initiation of suitable legal proceedings.

Copyright

Login ID:8FA03376/Student Name:ANSHULGARG/Overall Score:28

Question: 99
In the following questions, the words are coded according to a certain rule. Decode the rule and answer each question.

1)

3844549

2)

3844539

3)

3944539

4)

3944693

5)

None of these

ite
d

If ANDROID is coded as 1448594, then what will be the code for WINDOWS?

Li
m

Explanation:
ANDROID is coded as 1448594. Close observation reveals that the each letter is coded by the digit in the units place in the numerical
value. Hence, WINDOWS will be coded as 3944539. Hence, [3].

iv
at
e

Question: 100
In the following questions, the words are coded according to a certain rule. Decode the rule and answer each question.
If DHONI is coded as EIUOO, then what will be the code for KOHLI?
OUIOO

2)

OUIOU

3)

OUOOU

4)

OIOOU

5)

None of these

rc

es

Pr

1)

ou

Explanation:
Answer key: The respective coded letters are the first vowel after the letters in the original word. Hence, [1].

IM

Le

ar

ni

ng

es

Question: 101
Refer to the data below and answer the questions that follow.

For the input 12 20 MADE IN CHINA 65, which is final step?


1)

Step 3

2)

Step 4

3)

Step 5

4)

Step 6

5)

Step 2

IMS Learning Resources Pvt.Ltd.,Mumbai.All copyrights to this material vestswith IMS Learning Resources Pvt.Ltd.
No part of this materials either in part oras a whole shall be copied,printed,electronically reproduced,sold or distributed without the written
consent of IMS Learing Resources Pvt.Ltd.and any such violation would entail initiation of suitable legal proceedings.

Copyright

Login ID:8FA03376/Student Name:ANSHULGARG/Overall Score:28

Li
m

ite
d

Explanation:

iv
at
e

Step 3. Hence, [1].


Question: 102
Refer to the data below and answer the questions that follow.

CHINA

2)

20

3)

IN

4)

MADE

5)

12

ou

rc

es

1)

Pr

For the input "12 20 MADE IN CHINA 65",which word/number in the final step is at the same position as it was in the input?

IM

Le

ar

ni

ng

es

Explanation:

IN. Hence, [3].

Question: 103
Refer to the data below and answer the questions that follow.
For the input "12 20 MADE IN CHINA 65",which word/number in the final step has moved the farthest from its position in the input?
1)

CHINA

IMS Learning Resources Pvt.Ltd.,Mumbai.All copyrights to this material vestswith IMS Learning Resources Pvt.Ltd.
No part of this materials either in part oras a whole shall be copied,printed,electronically reproduced,sold or distributed without the written
consent of IMS Learing Resources Pvt.Ltd.and any such violation would entail initiation of suitable legal proceedings.

Copyright

Login ID:8FA03376/Student Name:ANSHULGARG/Overall Score:28

2)

20

3)

65

4)

MADE

5)

12

Pr

iv
at
e

Li
m

ite
d

Explanation:

65. Hence, [3].

rc

es

Question: 104
Refer to the data below and answer the questions that follow.

Step 4

3)

Step 5

4)

Step 6

5)

Step 7

es

2)

Step 3

ng

1)

ou

For the input "11 NORTH CATAPULT 18 56 VORTEX LEVEL 70", which is the final step?

IM

Le

ar

ni

Explanation:

IMS Learning Resources Pvt.Ltd.,Mumbai.All copyrights to this material vestswith IMS Learning Resources Pvt.Ltd.
No part of this materials either in part oras a whole shall be copied,printed,electronically reproduced,sold or distributed without the written
consent of IMS Learing Resources Pvt.Ltd.and any such violation would entail initiation of suitable legal proceedings.

Copyright

iv
at
e

Li
m

ite
d

Login ID:8FA03376/Student Name:ANSHULGARG/Overall Score:28

Step 6. Hence, [4].

es

Pr

Question: 105
Refer to the data below and answer the questions that follow.

56 CATAPULT 18 11 NORTH VORTEX LEVEL 70

2)

56 CATAPULT 11 NORTH 18 VORTEX LEVEL 70

3)

56 CATAPULT 18 LEVEL 11 NORTH VORTEX 70

4)

56 CATAPULT 18 LEVEL 70 11 NORTH VORTEX

5)

56 CATAPULT 18 LEVEL 70 NORTH 11 VORTEX

es

ou

1)

rc

For the input "11 NORTH CATAPULT 18 56 VORTEX LEVEL 70", which is the final output?

IM

Le

ar

ni

ng

Explanation:

IMS Learning Resources Pvt.Ltd.,Mumbai.All copyrights to this material vestswith IMS Learning Resources Pvt.Ltd.
No part of this materials either in part oras a whole shall be copied,printed,electronically reproduced,sold or distributed without the written
consent of IMS Learing Resources Pvt.Ltd.and any such violation would entail initiation of suitable legal proceedings.

Copyright

iv
at
e

Li
m

ite
d

Login ID:8FA03376/Student Name:ANSHULGARG/Overall Score:28

Hence, [5].

es

Pr

Question: 106
Refer to the data below and answer the questions that follow.

2)

CATAPULT

3)

56

4)

70

5)

18

ou

LEVEL

es

1)

rc

For the input "11 NORTH CATAPULT 18 56 VORTEX LEVEL 70", which of the following changes its position in Step 4 when compared
with Step 3?

IM

Le

ar

ni

ng

Explanation:

IMS Learning Resources Pvt.Ltd.,Mumbai.All copyrights to this material vestswith IMS Learning Resources Pvt.Ltd.
No part of this materials either in part oras a whole shall be copied,printed,electronically reproduced,sold or distributed without the written
consent of IMS Learing Resources Pvt.Ltd.and any such violation would entail initiation of suitable legal proceedings.

Copyright

iv
at
e

Li
m

ite
d

Login ID:8FA03376/Student Name:ANSHULGARG/Overall Score:28

LEVEL.Hence, [1].

es

Pr

Question: 107
Refer to the data below and answer the questions that follow.

rc

For the input "11 NORTH CATAPULT 18 56 VORTEX LEVEL 70", how many words/numbers retain their position in the final step as in the
input?
1

2)

3)

4)

5)

None of these

es

ou

1)

IM

Le

ar

ni

ng

Explanation:

IMS Learning Resources Pvt.Ltd.,Mumbai.All copyrights to this material vestswith IMS Learning Resources Pvt.Ltd.
No part of this materials either in part oras a whole shall be copied,printed,electronically reproduced,sold or distributed without the written
consent of IMS Learing Resources Pvt.Ltd.and any such violation would entail initiation of suitable legal proceedings.

Copyright

iv
at
e

Li
m

ite
d

Login ID:8FA03376/Student Name:ANSHULGARG/Overall Score:28

None of the words/numbers retain their position. Hence, [5].

es

ou

rc

es

Pr

Question: 108
Refer to the data below and answer the questions that follow.
In each of the following questions, symbols '$', '#', '@', '&' and '*' are used with different meanings as follows:
'A$B' means 'A is neither smaller than nor equal to B'
'A#B' means 'A is smaller than B'
'A@B' means 'A is not greater than B'
'A&B' means 'A is not smaller than B'
'A*B' means 'A is neither smaller than nor greater than B'

Le

ar

ni

ng

Now, in each of the following questions assuming the given statements to be true, find which of the two conclusions I and II given below
them is/are definitely true?
Give your answer as:
[1]: if only conclusion I is true
[2]: if only conclusion II is true
[3]: if either conclusion I or II is true
[4]: if neither conclusion I nor II is true
[5]: if both conclusions I and II are true

IM

Statements: F$H, E#G, F*E


Conclusion:
I. F#G
II.H@E
1)

2)

3)

4)

5)

IMS Learning Resources Pvt.Ltd.,Mumbai.All copyrights to this material vestswith IMS Learning Resources Pvt.Ltd.
No part of this materials either in part oras a whole shall be copied,printed,electronically reproduced,sold or distributed without the written
consent of IMS Learing Resources Pvt.Ltd.and any such violation would entail initiation of suitable legal proceedings.

Copyright

Login ID:8FA03376/Student Name:ANSHULGARG/Overall Score:28

Explanation:
We can build the following relation: G
Only conclusion[1] F

E=F

G is true. Hence, [1].

2)

3)

4)

5)

Li
m

iv
at
e

1)

ite
d

Question: 109
Refer to the data below and answer the questions that follow.
Statements: I * L, K # I, K $ J
Conclusion:
I.J@L
II. K@L

Explanation:

3)

4)

5)

es

es

2)

ng

1)

ou

Question: 110
Refer to the data below and answer the questions that follow.
Statements: M&P, O&N, O*M
Conclusion:
I.M*N
II. N # O

Pr

rc

We can build the following relation: L = I


K
Both the conclusions are not true. Hence, [4].

ni

Explanation:

ar

We can build the following relation: P


M=O
N
Either conclusionI is true or conclusionII is true. Hence, [3].

IM

Le

Question: 111
Refer to the data below and answer the questions that follow.
Statements: S@T, R&T, Q*S
Conclusion:
I. Q@T
II. R&Q
1)

2)

3)

4)

IMS Learning Resources Pvt.Ltd.,Mumbai.All copyrights to this material vestswith IMS Learning Resources Pvt.Ltd.
No part of this materials either in part oras a whole shall be copied,printed,electronically reproduced,sold or distributed without the written
consent of IMS Learing Resources Pvt.Ltd.and any such violation would entail initiation of suitable legal proceedings.

Copyright

Login ID:8FA03376/Student Name:ANSHULGARG/Overall Score:28

5)

Explanation:
We can build the following relation: R
T
Both the conclusions are true. Hence, [5].

S=Q

2)

3)

4)

5)

Li
m

iv
at
e

1)

ite
d

Question: 112
Refer to the data below and answer the questions that follow.
Statements: Y$W, X@Z, Z#Y
Conclusion:
I. X#Y
II. W*X

Explanation:
Y

3)

4)

5)

rc

es

2)

ng

ni

1)

ou

Question: 113
Refer to the data below and answer the questions that follow.
Statements: T$U, S$T, U#R
Conclusion:
I. T&R
II. S$U

es

Pr

We can build the following relation: W


Only conclusionI is true. Hence, [1].

Explanation:

Le

ar

We can build the following relation: R


Only conclusion II is true. Hence, [2].

IM

Question: 114
The statement below is followed by two courses of action. On the basis of the information given in the statement, choose the most
appropriate course of action.
Though all schools opened their doors to admit under-privileged kids under the RTE Act, many of their seats remained vacant.
Courses of action:
A. The government should start an advertising campaign to increase awareness about the RTE Act.
B. The admission process of all schools should be centralized.
1)

Only A

2)

Only B

3)

Both A and B

IMS Learning Resources Pvt.Ltd.,Mumbai.All copyrights to this material vestswith IMS Learning Resources Pvt.Ltd.
No part of this materials either in part oras a whole shall be copied,printed,electronically reproduced,sold or distributed without the written
consent of IMS Learing Resources Pvt.Ltd.and any such violation would entail initiation of suitable legal proceedings.

Copyright

Login ID:8FA03376/Student Name:ANSHULGARG/Overall Score:28

4)

Neither A nor B

5)

Either A or B

Explanation:
Only A is the correct course of action to be followed. If more people are aware of the RTE Act, then there would be an increase in the
number of people taking advantage of this act. B is a generalized statement and its impact cannot be ascertained.Hence, [1].

2)

3)

4)

5)

Pr

es

1)

iv
at
e

Li
m

ite
d

Question: 115
Each question is followed by two statements I and II. Answer each question using the following instructions.
Mark [1], if the question can be answered by using statement I alone.
Mark [2], if the question can be answered by using statement II alone.
Mark [3], if the question can be answered by using either of the statements alone.
Mark [4], if the question can be answered by using both the statements together, but cannot be answered by using either of the
statements alone.
Mark [5], if more data is required to answer the question.
How is 'the' coded in the given code language?
I. 'Lo Po Do Ko' is the code for 'Rajan stole the crown'.
II. 'Lo So Mo Ko Lo To Do' is the code for 'Is Rajan the thief of the crown'.

es

ou

rc

Explanation:
From statement I:
Statement I alone is not sufficient to answer the question as we have 4 words and 4 codes. It is not necessary that codes are in the same
order as words.
From statement II:
The word 'the' repeats twice and the code 'Lo' repeats twice. So the code corresponding to 'the' is 'Lo'. Therefore, Statement II alone is
sufficient to answer the question. Hence, [2].

IM

Le

ar

ni

ng

Question: 116
Each question is followed by two statements I and II. Answer each question using the following instructions.
Mark [1], if the question can be answered by using statement I alone.
Mark [2], if the question can be answered by using statement II alone.
Mark [3], if the question can be answered by using either of the statements alone.
Mark [4], if the question can be answered by using both the statements together, but cannot be answered by using either of the
statements alone.
Mark [5], if more data is required to answer the question.
Find the LCM of x and y, where x and y are natural numbers.
I.Product of x and y is 899.
II. y is 29.
1)

2)

3)

4)

5)

IMS Learning Resources Pvt.Ltd.,Mumbai.All copyrights to this material vestswith IMS Learning Resources Pvt.Ltd.
No part of this materials either in part oras a whole shall be copied,printed,electronically reproduced,sold or distributed without the written
consent of IMS Learing Resources Pvt.Ltd.and any such violation would entail initiation of suitable legal proceedings.

Copyright

Login ID:8FA03376/Student Name:ANSHULGARG/Overall Score:28

ite
d

Explanation:
From statement I: The values of x and y cannot be determined. But since x and y are natural numbers, if their product is 899, only two
cases are possible:
Case (i) One of them is 1 and other is 899.
Case (ii) One of them is 29 and other is 31.
In either case, the LCM of x and y = 899.
So, statement I is sufficient to give answer.
From statement II: Value of x is not known, so this statement alone is not sufficient to answer the question.
Hence, [1].

3)

4)

5)

rc

2)

ou

es

1)

es

Pr

iv
at
e

Li
m

Question: 117
Each question is followed by two statements I and II. Answer each question using the following instructions.
Mark [1], if the question can be answered by using statement I alone.
Mark [2], if the question can be answered by using statement II alone.
Mark [3], if the question can be answered by using either of the statements alone.
Mark [4], if the question can be answered by using both the statements together, but cannot be answered by using either of the
statements alone.
Mark [5], if more data is required to answer the question.

IM

Le

ar

ni

ng

Explanation:

IMS Learning Resources Pvt.Ltd.,Mumbai.All copyrights to this material vestswith IMS Learning Resources Pvt.Ltd.
No part of this materials either in part oras a whole shall be copied,printed,electronically reproduced,sold or distributed without the written
consent of IMS Learing Resources Pvt.Ltd.and any such violation would entail initiation of suitable legal proceedings.

Copyright

ou

rc

es

Pr

iv
at
e

Li
m

ite
d

Login ID:8FA03376/Student Name:ANSHULGARG/Overall Score:28

2)

3)

3
4

IM

4)

Le

1)

ar

ni

ng

es

Question: 118
Each question is followed by two statements I and II. Answer each question using the following instructions.
Mark [1], if the question can be answered by using statement I alone.
Mark [2], if the question can be answered by using statement II alone.
Mark [3], if the question can be answered by using either of the statements alone.
Mark [4], if the question can be answered by using both the statements together, but cannot be answered by using either of the
statements alone.
Mark [5], if more data is required to answer the question.
In an election, all 900 voters voted either for Alfredo or Ernesto (but not both). What percentage of female voters voted for Alfredo?
I.Eighty percent of the female voters voted for Ernesto.
II.Seventy percent of the voters voted for Alfredo.

5)

Explanation:
Statement I alone is sufficient to answer the question as we can conclude that 20% of the female voted for Alfredo.
Statement II alone is not sufficient to answer the question. Hence, [1].
Question: 119

IMS Learning Resources Pvt.Ltd.,Mumbai.All copyrights to this material vestswith IMS Learning Resources Pvt.Ltd.
No part of this materials either in part oras a whole shall be copied,printed,electronically reproduced,sold or distributed without the written
consent of IMS Learing Resources Pvt.Ltd.and any such violation would entail initiation of suitable legal proceedings.

Copyright

Login ID:8FA03376/Student Name:ANSHULGARG/Overall Score:28

Refer to the data below and answer the questions that follow.
In the year 2008, a Agro-based company sold 20000 sprinklers and generated total sales of Rs.15 crores. Its total sales increased by 5
crores every year for the next two years, whereas the number of sprinklers sold increased by 20% every year over the previous year.

1)

20000

2)

22000

3)

24000

4)

28000

5)

29000

ite
d

What is the number of sprinklers sold in year 2009?

ou

rc

es

Pr

iv
at
e

Li
m

Explanation:

es

Hence, [3].

Question: 120
Refer to the data below and answer the questions that follow.

ng

What is the percentage increase in the total revenue in the year 2009 over the previous year?
20%

2)

25%

3)

50%

4)

33.33%

5)

35%

ar

Le

IM

Explanation:

ni

1)

IMS Learning Resources Pvt.Ltd.,Mumbai.All copyrights to this material vestswith IMS Learning Resources Pvt.Ltd.
No part of this materials either in part oras a whole shall be copied,printed,electronically reproduced,sold or distributed without the written
consent of IMS Learing Resources Pvt.Ltd.and any such violation would entail initiation of suitable legal proceedings.

Copyright

Pr

iv
at
e

Li
m

ite
d

Login ID:8FA03376/Student Name:ANSHULGARG/Overall Score:28

es

Question: 121
Refer to the data below and answer the questions that follow.

What is the number of sprinklers sold in 2011, if the percentage increase in the total units sold in 2011 as compared to 2010 is 20%?
34560

2)

34410

3)

34840

4)

35020

5)

None of these

es

ou

rc

1)

IM

Le

ar

ni

ng

Explanation:

IMS Learning Resources Pvt.Ltd.,Mumbai.All copyrights to this material vestswith IMS Learning Resources Pvt.Ltd.
No part of this materials either in part oras a whole shall be copied,printed,electronically reproduced,sold or distributed without the written
consent of IMS Learing Resources Pvt.Ltd.and any such violation would entail initiation of suitable legal proceedings.

Copyright

Login ID:8FA03376/Student Name:ANSHULGARG/Overall Score:28

Question: 122
Refer to the data below and answer the questions that follow.

Rs.7500

2)

Rs.8333

3)

Rs.10000

4)

Rs.12333

5)

None of these

iv
at
e

1)

Li
m

ite
d

What is the average cost of one sprinkler sold in the year 2009?

es

ou

rc

es

Pr

Explanation:

ng

Rs.8333. Hence, [2].

ni

Question: 123
Refer to the data below and answer the questions that follow.

2)

Rs.8680

3)

Rs.9000

4)

ar

Rs.7500

Le

1)

What is the average cost of one sprinkler sold in the year 2010?

IM

Rs.9500

5)

Rs.8200

Explanation:

IMS Learning Resources Pvt.Ltd.,Mumbai.All copyrights to this material vestswith IMS Learning Resources Pvt.Ltd.
No part of this materials either in part oras a whole shall be copied,printed,electronically reproduced,sold or distributed without the written
consent of IMS Learing Resources Pvt.Ltd.and any such violation would entail initiation of suitable legal proceedings.

Copyright

Li
m

ite
d

Login ID:8FA03376/Student Name:ANSHULGARG/Overall Score:28

iv
at
e

Hence, [2].

Mark [1], if A

Mark [2], if B

Mark [3], if A

Pr

Question: 124
Using the information given in the question, compute the values of A and B.

rc

es

Mark [4], if B
A
Mark [5], if A = B or if no relation exists

3)

4)

5)

es

2)

ng

1)

ou

A4= 4096
B3= 729

Le

ar

ni

Explanation:

IM

Question: 125
Using the information given in the question, compute the values of A and B.
Mark [1], if A

Mark [2], if B

Mark [3], if A

Mark [4], if B
A
Mark [5], if A = B or if no relation exists

IMS Learning Resources Pvt.Ltd.,Mumbai.All copyrights to this material vestswith IMS Learning Resources Pvt.Ltd.
No part of this materials either in part oras a whole shall be copied,printed,electronically reproduced,sold or distributed without the written
consent of IMS Learing Resources Pvt.Ltd.and any such violation would entail initiation of suitable legal proceedings.

Copyright

Login ID:8FA03376/Student Name:ANSHULGARG/Overall Score:28

A232A + 255 = 0
B228B + 195 = 0
1)

2)

3)

4)

5)

iv
at
e

Li
m

ite
d

Explanation:

Mark [2], if B

Mark [3], if A

es

Mark [1], if A

Pr

Question: 126
Using the information given in the question, compute the values of A and B.

3)

4)

5)

es

2)

ng

ni

1)

ou

rc

Mark [4], if B
A
Mark [5], if A = B or if no relation exists
A: Radius of the sphere with volume 38808 cm3
B: Radius of the cylinder with volume 29106 cm3, where the radius of the cylinder is equal to its height

IM

Le

ar

Explanation:

Question: 127

IMS Learning Resources Pvt.Ltd.,Mumbai.All copyrights to this material vestswith IMS Learning Resources Pvt.Ltd.
No part of this materials either in part oras a whole shall be copied,printed,electronically reproduced,sold or distributed without the written
consent of IMS Learing Resources Pvt.Ltd.and any such violation would entail initiation of suitable legal proceedings.

Copyright

Login ID:8FA03376/Student Name:ANSHULGARG/Overall Score:28

Using the information given in the question, compute the values of A and B.
Mark [1], if A

Mark [2], if B

Mark [3], if A

2)

3)

4)

5)

Li
m

iv
at
e

1)

ite
d

Mark [4], if B
A
Mark [5], if A = B or if no relation exists
A: Common difference in an arithmetic progression of 10 terms, where the 1st term is 3 and the sum of all theterms of the progression is
165.
B:Common ratio in a geometric progression of 7 terms, where the first term is 3 and the 7th term is 2187.

es

ou

rc

es

Pr

Explanation:

Mark [2], if B

Mark [3], if A

ng

ni

Mark [1], if A

Question: 128
Using the information given in the question, compute the values of A and B.

1)

2)

Le

ar

Mark [4], if B
A
Mark [5], if A = B or if no relation exists
Given A and B are natural numbers.
3A + 4B = 11

IM

3)

4)

5)

Explanation:

IMS Learning Resources Pvt.Ltd.,Mumbai.All copyrights to this material vestswith IMS Learning Resources Pvt.Ltd.
No part of this materials either in part oras a whole shall be copied,printed,electronically reproduced,sold or distributed without the written
consent of IMS Learing Resources Pvt.Ltd.and any such violation would entail initiation of suitable legal proceedings.

Copyright

Login ID:8FA03376/Student Name:ANSHULGARG/Overall Score:28

Mark [2], if B

Mark [3], if A

2)

3)

4)

5)

Pr

es

1)

iv
at
e

Mark [4], if B
A
Mark [5], if A = B or if no relation exists
2 six sided unbiased dice are rolled
A: The probability that the sum of the numbers on the 2 dice is the square of a number.
B: The probability that the product of the numbers on the 2 dice is the square of anumber.

Li
m

Mark [1], if A

ite
d

Question: 129
Using the information given in the question, compute the values of A and B.

IM

Le

ar

ni

ng

es

ou

rc

Explanation:

Question: 130
Using the information given in the question, compute the values of A and B.
Mark [1], if A

IMS Learning Resources Pvt.Ltd.,Mumbai.All copyrights to this material vestswith IMS Learning Resources Pvt.Ltd.
No part of this materials either in part oras a whole shall be copied,printed,electronically reproduced,sold or distributed without the written
consent of IMS Learing Resources Pvt.Ltd.and any such violation would entail initiation of suitable legal proceedings.

Copyright

Login ID:8FA03376/Student Name:ANSHULGARG/Overall Score:28

Mark [2], if B

Mark [3], if A

2)

3)

4)

5)

Li
m

1)

ite
d

Mark [4], if B
A
Mark [5], if A = B or if no relation exists
A: Number of ways in which 6 people can sit around a circular table
B: Number of ways in which 5 people can be arranged in a line

Pr

Question: 131
Choose the correct alternative.

iv
at
e

Explanation:
Number of possibilities of circular arrangement of 6 people = (6 1)! = 120.
Number of possibilities of linear arrangement of 5 people = 5! = 120.
Therefore, A = B. Hence, [5].

1100 litres

3)

1050 litres

4)

2200 litres

5)

1025 litres

ou

2)

es

2100 litres

1)

rc

es

Four taps A, B, C, D are connected to a tank. Taps A and B are used as inlets, while taps C and D are used as outlets. A and B alone fill
the tank in 20 minutes and 35 minutes respectively; C empties the tank in 25 minutes, and the flow rate of D is 23 litres/min. If all the taps
are opened, the tank gets filled in one hour. What is the capacity of the tank?

IM

Le

ar

ni

ng

Explanation:

IMS Learning Resources Pvt.Ltd.,Mumbai.All copyrights to this material vestswith IMS Learning Resources Pvt.Ltd.
No part of this materials either in part oras a whole shall be copied,printed,electronically reproduced,sold or distributed without the written
consent of IMS Learing Resources Pvt.Ltd.and any such violation would entail initiation of suitable legal proceedings.

Copyright

Login ID:8FA03376/Student Name:ANSHULGARG/Overall Score:28

Question: 132
Choose the correct alternative.

1)

80 m

2)

20 m

3)

140 m

4)

100 m

5)

Cannot be determined

ite
d

Train A crosses train B in 30 seconds when travelling in the same direction, and 5 seconds when travelling in the opposite direction. Find
the length of train A, if train B is 100 m long.

es

ou

rc

es

Pr

iv
at
e

Li
m

Explanation:

ng

Question: 133
Choose the correct alternative.

Rs.44

2)

Rs.48

3)

Rs.50

4)

Rs.62

Cannot be determined

IM

5)

Le

1)

ar

ni

The ratio of the cost price of cookie A to that of cookie B is 2 : 5. The ratio of profits made on selling the cookies is in the ratio 4 : 5. If the
selling price of cookie B is Rs.100, find the selling price of cookie A.

Explanation:
Let the cost price for cookies A and B be 2x and 5x respectively, and the profits for cookies A and B be 4y and 5y respectively.
Thus, for cookie B, 5x + 5y = 100, or x + y =20and for cookie A, 2x + 4y is the S.P.
But we do not know the value of x and y hence the selling price of cookie A cannot be determined.
Hence, [5].
Question: 134

IMS Learning Resources Pvt.Ltd.,Mumbai.All copyrights to this material vestswith IMS Learning Resources Pvt.Ltd.
No part of this materials either in part oras a whole shall be copied,printed,electronically reproduced,sold or distributed without the written
consent of IMS Learing Resources Pvt.Ltd.and any such violation would entail initiation of suitable legal proceedings.

Copyright

Login ID:8FA03376/Student Name:ANSHULGARG/Overall Score:28

2)

6 104

3)

26104

4)

(104)6

5)

Cannot be determined

ite
d

Choose the correct alternative.


A certain population of bacteria doubles every 10 minutes. If the number of bacteria in the population initially was 104, then what was the
population 1 hour later?
1)
2 104

iv
at
e

Li
m

Explanation:

Pr

Question: 135
Choose the correct alternative.

22 ml

3)

30 ml

4)

36 ml

5)

48 ml

rc

2)

ou

20 ml

es

1)

es

A sugar syrup contains 78% sugar and the rest is water. On heating the syrup, 18 ml water evaporates and 282 ml of the syrup is left.
How much water is present in the syrup now?

ar

ni

ng

Explanation:

Le

Question: 136
Choose the correct alternative.

IM

Find the number of ways of arranging 4 mathematics, 3 physics and 2 chemistry books such that no two Physics books are together.
(Assume each of the 9 books are distinct)
6! 7P3
1)
2)

7! 6C3

3)

7P

4)

IMS Learning Resources Pvt.Ltd.,Mumbai.All copyrights to this material vestswith IMS Learning Resources Pvt.Ltd.
No part of this materials either in part oras a whole shall be copied,printed,electronically reproduced,sold or distributed without the written
consent of IMS Learing Resources Pvt.Ltd.and any such violation would entail initiation of suitable legal proceedings.

Copyright

Login ID:8FA03376/Student Name:ANSHULGARG/Overall Score:28

5)

None of these

ite
d

Explanation:

ou

rc

es

Pr

iv
at
e

Li
m

Question: 137
Answer the questions on the basis of the diagram given below.

es

What is the length of BE (in cm)?

ng

1)

ar

ni

2)

5)

None of these

IM

4)

Le

3)

Explanation:

IMS Learning Resources Pvt.Ltd.,Mumbai.All copyrights to this material vestswith IMS Learning Resources Pvt.Ltd.
No part of this materials either in part oras a whole shall be copied,printed,electronically reproduced,sold or distributed without the written
consent of IMS Learing Resources Pvt.Ltd.and any such violation would entail initiation of suitable legal proceedings.

Copyright

iv
at
e

Li
m

ite
d

Login ID:8FA03376/Student Name:ANSHULGARG/Overall Score:28

Pr

Question: 138
Answer the questions on the basis of the diagram given below.

es

What is the area of quadrilateral OABC (in sq cm)?

rc

1)

es

ou

2)

3)

5)

None of these

IM

Le

ar

ni

Explanation:

ng

4)

IMS Learning Resources Pvt.Ltd.,Mumbai.All copyrights to this material vestswith IMS Learning Resources Pvt.Ltd.
No part of this materials either in part oras a whole shall be copied,printed,electronically reproduced,sold or distributed without the written
consent of IMS Learing Resources Pvt.Ltd.and any such violation would entail initiation of suitable legal proceedings.

Copyright

ite
d

Login ID:8FA03376/Student Name:ANSHULGARG/Overall Score:28

2)

128 sq. cm

3)

256 sq. cm

4)

512 sq. cm

5)

None of these

iv
at
e

64 sq. cm

Pr

1)

Li
m

Question: 139
Answer the questions on the basis of the diagram given below.

ni

ng

es

ou

rc

es

Explanation:

IM

Le

ar

Question: 140
Refer to the data below and answer the questions that follow.

IMS Learning Resources Pvt.Ltd.,Mumbai.All copyrights to this material vestswith IMS Learning Resources Pvt.Ltd.
No part of this materials either in part oras a whole shall be copied,printed,electronically reproduced,sold or distributed without the written
consent of IMS Learing Resources Pvt.Ltd.and any such violation would entail initiation of suitable legal proceedings.

Copyright

Login ID:8FA03376/Student Name:ANSHULGARG/Overall Score:28

Find the number of instances where the scarcity of wheat is more than 100 crore tons in a particular year for a particular country.
1)

2)

3)

4)

5)

iv
at
e

Li
m

ite
d

Explanation:

Pr

The scarcity of wheat is more than 100 crore tons in Brazil in 2014 and India in 2011 and 2012. Hence, [4].

es

Question: 141
Refer to the data below and answer the questions that follow.

6%

3)

8%

4)

9%

5)

10%

ou

2)

es

3%

1)

rc

What is the compounded annual percentage growth in wheat production of Brazil over the given period?

IM

Le

ar

ni

ng

Explanation:

IMS Learning Resources Pvt.Ltd.,Mumbai.All copyrights to this material vestswith IMS Learning Resources Pvt.Ltd.
No part of this materials either in part oras a whole shall be copied,printed,electronically reproduced,sold or distributed without the written
consent of IMS Learing Resources Pvt.Ltd.and any such violation would entail initiation of suitable legal proceedings.

Copyright

Login ID:8FA03376/Student Name:ANSHULGARG/Overall Score:28

ite
d

Question: 142
Refer to the data below and answer the questions that follow.

110 crore tons scarcity

2)

110 crore tons surplus

3)

100 crore tons surplus

4)

75 crore tons scarcity

5)

75 crore tons surplus

iv
at
e

1)

Li
m

What is the average annual wheat scarcity/surplus in India for the given period?

ar

ni

ng

es

ou

rc

es

Pr

Explanation:

Le

Question: 143
Refer to the data below and answer the questions that follow.
What is the approximate percentage change in scarcity/surplus of wheat in the following countries over the given period?
173%

2)

1)

3)

163%

4)

153%

5)

91%

IM

44%

Explanation:

IMS Learning Resources Pvt.Ltd.,Mumbai.All copyrights to this material vestswith IMS Learning Resources Pvt.Ltd.
No part of this materials either in part oras a whole shall be copied,printed,electronically reproduced,sold or distributed without the written
consent of IMS Learing Resources Pvt.Ltd.and any such violation would entail initiation of suitable legal proceedings.

Copyright

Pr

iv
at
e

Li
m

ite
d

Login ID:8FA03376/Student Name:ANSHULGARG/Overall Score:28

es

Question: 144
Refer to the data below and answer the questions that follow.

18.8%

3)

22.6%

4)

20.6%

5)

23.2%

ou

2)

es

19.8%

1)

rc

What is the percentage change in production of wheat in India over the given period?

IM

Le

ar

ni

ng

Explanation:

Question: 145
Refer to the data below and answer the questions that follow.

IMS Learning Resources Pvt.Ltd.,Mumbai.All copyrights to this material vestswith IMS Learning Resources Pvt.Ltd.
No part of this materials either in part oras a whole shall be copied,printed,electronically reproduced,sold or distributed without the written
consent of IMS Learing Resources Pvt.Ltd.and any such violation would entail initiation of suitable legal proceedings.

Copyright

Li
m

ite
d

Login ID:8FA03376/Student Name:ANSHULGARG/Overall Score:28

9.45 lakhs

2)

9.8 lakhs

3)

0.98 lakhs

4)

1.1 million

5)

7.7 lakhs

Pr

1)

iv
at
e

In the year 2010, what were the total number of cars manufactured by companies other than the given 6 companies?

ou

rc

es

Explanation:

es

Question: 146
Refer to the data below and answer the questions that follow.

In the year 2010, by what percent is the leading car manufacturer ahead of the manufacturer of the least number of cars?
200%

2)

10%

3)

300%

4)

100%

5)

Cannot be determined

Le

ar

ni

ng

1)

Explanation:
It is not possible to determine the leading car manufacturer as wellas the least car manufactured in the year 2010. Thus the question
cannot be answered.Hence, [5].

IM

Question: 147
Refer to the data below and answer the questions that follow.
What is the percentage increase in VW's production capacity from 2010 to 2014?
1)

44%

2)

78%

3)

70%

IMS Learning Resources Pvt.Ltd.,Mumbai.All copyrights to this material vestswith IMS Learning Resources Pvt.Ltd.
No part of this materials either in part oras a whole shall be copied,printed,electronically reproduced,sold or distributed without the written
consent of IMS Learing Resources Pvt.Ltd.and any such violation would entail initiation of suitable legal proceedings.

Copyright

Login ID:8FA03376/Student Name:ANSHULGARG/Overall Score:28

4)

86%

5)

92%

ite
d

Explanation:

Li
m

Question: 148
Refer to the data below and answer the questions that follow.

1 million

2)

0.2 million

3)

0.175 million

4)

0.25 million

5)

0.225 million

Pr

1)

iv
at
e

How many cars did PEUGEOT manufacture in the year 2014?

rc

ou

Question: 149
Refer to the data below and answer the questions that follow.

es

Explanation:
Cars manufactured = 10% of 2.5 million = 0.25 million. Hence, [4].

110%

3)

143%

4)

190%

5)

93%

2)

ng

240%

ni

1)

es

What is the percentage increase in the number of cars manufactured by the 6 given companies from 2010 to 2014?

IM

Le

ar

Explanation:

Question: 150
Refer to the bar graph below and answer the questions that follow.

IMS Learning Resources Pvt.Ltd.,Mumbai.All copyrights to this material vestswith IMS Learning Resources Pvt.Ltd.
No part of this materials either in part oras a whole shall be copied,printed,electronically reproduced,sold or distributed without the written
consent of IMS Learing Resources Pvt.Ltd.and any such violation would entail initiation of suitable legal proceedings.

Copyright

Pr

iv
at
e

Li
m

ite
d

Login ID:8FA03376/Student Name:ANSHULGARG/Overall Score:28

165000

3)

125000

4)

140000

5)

150000

rc

2)

ou

90000

es

1)

es

If the product supply for the year 2014 is 65000 tons, the capacity of the product then is:

Le

ar

ni

ng

Explanation:

IM

Question: 151
Refer to the bar graph below and answer the questions that follow.
If the capacity of the industry to produce the product remains constant at 1 lakh tons, what is the average supply for the given period?
1)

53750

2)

75300

3)

57730

4)

55350

IMS Learning Resources Pvt.Ltd.,Mumbai.All copyrights to this material vestswith IMS Learning Resources Pvt.Ltd.
No part of this materials either in part oras a whole shall be copied,printed,electronically reproduced,sold or distributed without the written
consent of IMS Learing Resources Pvt.Ltd.and any such violation would entail initiation of suitable legal proceedings.

Copyright

Login ID:8FA03376/Student Name:ANSHULGARG/Overall Score:28

5)

67500

iv
at
e

Li
m

ite
d

Explanation:

Question: 152
Refer to the bar graph below and answer the questions that follow.

Industry capacity steadily increases during the given period

2)

Industry capacity steadily decreases during the given period

3)

Industry capacity fluctuates during the given period

4)

Industry capacity remains the same during the given period

5)

None of these

ou

rc

es

1)

Pr

If the product supply remains unchanged for the given period, then what does the trend of industry capacity signify?

IM

Le

ar

ni

ng

es

Explanation:

Question: 153
Refer to the bar graph below and answer the questions that follow.
Which year shows the highest increase in the difference between the industry capacity and supply over the previous year, if demand for
the product is 80000 tons for each of the given years?
1)

2011

IMS Learning Resources Pvt.Ltd.,Mumbai.All copyrights to this material vestswith IMS Learning Resources Pvt.Ltd.
No part of this materials either in part oras a whole shall be copied,printed,electronically reproduced,sold or distributed without the written
consent of IMS Learing Resources Pvt.Ltd.and any such violation would entail initiation of suitable legal proceedings.

Copyright

Login ID:8FA03376/Student Name:ANSHULGARG/Overall Score:28

2)

2012

3)

2013

4)

2014

5)

Cannot be determined

ou

Question: 154
Refer to the bar graph below and answer the questions that follow.

rc

es

Pr

iv
at
e

Li
m

ite
d

Explanation:

es

In 2013, if the industry capacity is 20000 tons more than twice of the product supply, then what is the product supply (approximately) for
the year 2013?
240000

2)

182100

3)

222100

4)

180000

5)

None of these

ar

ni

ng

1)

IM

Le

Explanation:

Question: 155
Choose the correct alternative.
The third term and the seventh term of an A.P. are 11 and 19 respectively. Find the fifth term.

IMS Learning Resources Pvt.Ltd.,Mumbai.All copyrights to this material vestswith IMS Learning Resources Pvt.Ltd.
No part of this materials either in part oras a whole shall be copied,printed,electronically reproduced,sold or distributed without the written
consent of IMS Learing Resources Pvt.Ltd.and any such violation would entail initiation of suitable legal proceedings.

Copyright

Login ID:8FA03376/Student Name:ANSHULGARG/Overall Score:28

1)
2)
3)

15

4)

18

5)

25

ite
d

Explanation:
The fifth term will be the average of the third and seventh terms. Hence, [3].

Li
m

Question: 156
Choose the correct alternative.

The average runs made by the first 6 batsmen in the batting order is 40 and the corresponding average for the rest 5 batsmen is 32. Find
the combined average.
36.36

2)

37.5

3)

35

4)

32.8

5)

None of these

Pr

iv
at
e

1)

ou

rc

es

Explanation:

es

Question: 157
Choose the correct alternative.

3)

50%

4)

60%

5)

45%

IM

Explanation:

ni

40%

ar

2)

Le

55%

1)

ng

In an examination, 35% of the total students failed in Maths, 45% failed in Science and 30% failed in both. Find the percentage of students
who passed in both subjects.

IMS Learning Resources Pvt.Ltd.,Mumbai.All copyrights to this material vestswith IMS Learning Resources Pvt.Ltd.
No part of this materials either in part oras a whole shall be copied,printed,electronically reproduced,sold or distributed without the written
consent of IMS Learing Resources Pvt.Ltd.and any such violation would entail initiation of suitable legal proceedings.

Copyright

ite
d

Login ID:8FA03376/Student Name:ANSHULGARG/Overall Score:28

2)

20 m and 14 m

3)

12 m and 8 m

4)

24 m and 14 m

5)

16 m and 10 m

iv
at
e

16 m and 12 m

Pr

1)

Li
m

Question: 158
Choose the correct alternative.
The length and breadth of a rectangular room, each when increased by 4 m, are in the ratio 4 : 3, and each when decreased by 4 m, are
in the ratio 2 : 1. The original dimensions of the room are:

ni

IM

Le

ar

Question: 159
Choose the correct alternative.

ng

es

ou

rc

es

Explanation:

1)

2)

3)

IMS Learning Resources Pvt.Ltd.,Mumbai.All copyrights to this material vestswith IMS Learning Resources Pvt.Ltd.
No part of this materials either in part oras a whole shall be copied,printed,electronically reproduced,sold or distributed without the written
consent of IMS Learing Resources Pvt.Ltd.and any such violation would entail initiation of suitable legal proceedings.

Copyright

Login ID:8FA03376/Student Name:ANSHULGARG/Overall Score:28

4)

5)

Li
m

ite
d

Explanation:

iv
at
e

Question: 160
Choose the correct alternative.

20 days

3)

30 days

4)

45 days

5)

60 days

es

2)

rc

15 days

ou

1)

Pr

Nisha and Archana complete a work together in 10 days. If Nisha can do the work alone in 12 days, find how much time will Archana take
to complete the work alone.

ng

es

Explanation:

ar

ni

Question: 161
Four words are highlighted in each sentence. Identify the one that is inappropriately/incorrectly used.

Le

While the Commonwealth Games brought with them their own fare share of shame, athletes say that even badly organized international
events leave an infrastructure legacy of some sort.
brought

2)

fare

IM

1)

3)

even

4)

legacy

5)

None of the above

Explanation:
'Fare' is a noun which means 'something offered to the public for entertainment' or 'the price of conveyance in a vehicle'. The correct word
here will be 'fair', an adjective that means 'moderately large' or 'ample'. Hence, [2].

IMS Learning Resources Pvt.Ltd.,Mumbai.All copyrights to this material vestswith IMS Learning Resources Pvt.Ltd.
No part of this materials either in part oras a whole shall be copied,printed,electronically reproduced,sold or distributed without the written
consent of IMS Learing Resources Pvt.Ltd.and any such violation would entail initiation of suitable legal proceedings.

Copyright

Login ID:8FA03376/Student Name:ANSHULGARG/Overall Score:28

Question: 162
Four words are highlighted in each sentence. Identify the one that is inappropriately/incorrectly used.

1)

silence

2)

from

3)

bearing

4)

these

5)

None of the above

ite
d

The use of Russian language in Central Asia highlights the silence of Russian language use in the countries separated from the former
Soviet Union and has a direct bearing on the security of these countries.

Li
m

Explanation:
'Silence' doesn't make any sense in this context. However, 'salience', which means 'prominence', is the right word to be used in this
context. Hence, [1].

iv
at
e

Question: 163
Four words are highlighted in each sentence. Identify the one that is inappropriately/incorrectly used.

Pr

Kazakhstan experienced the effects of the global economic downturn, but averted economic disaster with a policy deigned to protect the
country and its people from the economic crisis.
experienced

2)

averted

3)

deigned

4)

crisis

5)

None of the above

ou

rc

es

1)

es

Explanation:
A policy cannot be 'deigned' but it can be 'designed'. 'Deigned' means 'condescended to give or grant' while 'designed' means 'intended
for a definite purpose'.Hence, [3].

ng

Question: 164
Four words are highlighted in each sentence. Identify the one that is inappropriately/incorrectly used.

2)

clamps

3)

dissent

4)

kindle

None of the above

IM

5)

ar

regimes

Le

1)

ni

Central Asian authoritarian regimes, fixated on preserving their rule, clamps down on civil society and political dissent, and in doing so
they kindle further opposition among their citizens.

Explanation:
The subject of the sentence is 'regimes', which is a plural noun. Thus, the singular form of the verb 'clamps' is incorrect in the given
context. 'Clamp', which is its plural form, would be correct. Hence, [2].
Question: 165
Four words are highlighted in each sentence. Identify the one that is inappropriately/incorrectly used.

IMS Learning Resources Pvt.Ltd.,Mumbai.All copyrights to this material vestswith IMS Learning Resources Pvt.Ltd.
No part of this materials either in part oras a whole shall be copied,printed,electronically reproduced,sold or distributed without the written
consent of IMS Learing Resources Pvt.Ltd.and any such violation would entail initiation of suitable legal proceedings.

Copyright

Login ID:8FA03376/Student Name:ANSHULGARG/Overall Score:28

1)

During

2)

backgrounds

3)

maintain

4)

their

5)

None of the above

Explanation:
There is no error in the given sentence. Hence, [5].

Li
m

Question: 166
Each sentence has a highlighted phrase. Replace the phrase with the correct option from the choices given below:

2)

blown with

3)

blown away by

4)

blown

5)

blown away

Pr

blown way at

iv
at
e

As all humorists must be, I was totally blown way at my own wit.
1)

ite
d

During the summer, children with rural family backgrounds return to the steppe to help their families maintain the herds, and some
come back to live in the steppe after finishing their education.

ou

rc

es

Explanation:
The correct phrase is 'blown away' (meaning 'was extremely impressed'). The correct usage is 'blown away by', which is followed by the
thing that impresses. Hence, [3].

es

Question: 167
Each sentence has a highlighted phrase. Replace the phrase with the correct option from the choices given below:

mystic being

3)

mysterious being

4)

master of being

5)

mystical of being

ng

2)

ni

mystery of being

ar

1)

When we begin to realize this, we begin to participate in the mystery of being that is so central to our existence.

Le

Explanation:
'Mystery of being' is the correct phrase. The adjectival phrase 'of being' describes the noun 'mystery' perfectly. Thus, the sentence is
correct and no change is required. Hence, [1].

IM

Question: 168
Each sentence has a highlighted phrase. Replace the phrase with the correct option from the choices given below:
This meeting will be a sheer waste of resources and time if we do not get down to the brass stacks soon enough.
1)

get down to the brass stacks

2)

get down to the bass tack

3)

get down to brass stakes

IMS Learning Resources Pvt.Ltd.,Mumbai.All copyrights to this material vestswith IMS Learning Resources Pvt.Ltd.
No part of this materials either in part oras a whole shall be copied,printed,electronically reproduced,sold or distributed without the written
consent of IMS Learing Resources Pvt.Ltd.and any such violation would entail initiation of suitable legal proceedings.

Copyright

Login ID:8FA03376/Student Name:ANSHULGARG/Overall Score:28

4)

get over with brassy stacks

5)

get down to brass tacks

Explanation:
'Get down to brass tacks' is the correct usage, which means 'get to the real issue'. All the other phrases are incorrect. Hence, [5].
Question: 169
Each sentence has a highlighted phrase. Replace the phrase with the correct option from the choices given below:

2)

with the speed but on the scale

3)

with the speed or on the scale

4)

by the speed or the scale

5)

on the speed with the scale

Li
m

with the speed or the scale

iv
at
e

1)

ite
d

World leaders have failed to act with the speed or the scale required to protect citizens from potential catastrophe.

Pr

Explanation:
Here, the two phrases 'with the speed' and 'on the scale' imply two different things and can be connected with 'or' as long as the individual
prepositions ('with' and 'on') are retained. Hence, [3].

es

Question: 170
Each sentence has a highlighted phrase. Replace the phrase with the correct option from the choices given below:

pass through

2)

pass off

3)

pass around

4)

pass on

5)

pass up

es

ou

1)

rc

There are certain personality traits and physical characteristics that parents pass through to their children.

ni

ng

Explanation:
'Pass through' means 'move through'; 'pass off' means 'present deceptively'; 'pass around' means 'circulate'; 'pass up' means 'let go
without accepting'. Thus, 'pass on', which can imply 'transmit', is the most apt phrase. Hence, [4].

IM

Le

ar

Question: 171
The following paragraph has some words missing. Choose the options that best fill the blanks.
Black Beauty, a shiny, scaly-skinned, 4.4-billion-year-old rock from Mars, has arguably become the most important ___(11)___ meteorite
among the 75 that have fallen to Earth. At more than $10,000 per gram, it is certainly one of the most expensive. Scientists ___(12)___
the meteorite not only because it is the oldest meteorite from Mars, but also because it appears to be the first sedimentary one a rock
made of pieces that ___(13)___ eroded, transported and deposited by wind or water. The meteorite is a breccia a rock made of smaller
rocks, fused in a fine-grained matrix and so each of the embedded pebbles has a story to tell. But Black Beauty's story on Earth from
the nomad who found it in the dunes of the western Sahara to the wealthy weight loss doctor who now owns most of the meteorite is
also important, because it ___(14)___ the ___(15)___ relationship between meteorite hunters, collectors and scientists.
1] minor
2] Martian
3] magnetic
4] martial
5] manned
1)

IMS Learning Resources Pvt.Ltd.,Mumbai.All copyrights to this material vestswith IMS Learning Resources Pvt.Ltd.
No part of this materials either in part oras a whole shall be copied,printed,electronically reproduced,sold or distributed without the written
consent of IMS Learing Resources Pvt.Ltd.and any such violation would entail initiation of suitable legal proceedings.

Copyright

Login ID:8FA03376/Student Name:ANSHULGARG/Overall Score:28

2)

3)

4)

5)

2)

3)

4)

5)

Li
m

Pr

es

1)

iv
at
e

Question: 172
The following paragraph has some words missing. Choose the options that best fill the blanks.
1] recognize
2] merit
3] value
4] peg
5] count

ite
d

Explanation:
'Minor' cannot follow 'important' and 'martial' is completely unrelated to the context. A meteorite cannot be 'manned'. 'Magnetic' could be
the feature of a rock but not a classification. Since the sentence discusses an old rock from Mars, the most appropriate choice will be a
word pertaining to Mars, i.e. 'Martian'. Hence, [2].

es

ou

rc

Explanation:
Since the positive aspects of the meteorite are discussed, scientists will 'value' those distinguishing features. 'Recognise' means to
'acknowledge', and it is a milder word for such an expensive rock. 'Peg' and 'count' are not what scientists will do in this context. 'Merit'
means 'deserve' and is incorrect here as its subject should be 'meteorite', not 'scientists'. So, 'value' is the best fit. Hence, [3].

3)

4)

4
5

IM

5)

ar

2)

Le

1)

ni

ng

Question: 173
The following paragraph has some words missing. Choose the options that best fill the blanks.
1] was
2] were
3] had to be
4] would be
5] would get

Explanation:
Here, the process of rock formation is in the past. So, 'would get' and 'would be' will not fit. Also, the word should agree with 'pieces' and
not 'rock'. So, 'was' can be ruled out. The process of erosion was not intended by someone, thus 'had to be' also does not fit in the
context. Hence, [2].
Question: 174
The following paragraph has some words missing. Choose the options that best fill the blanks.

IMS Learning Resources Pvt.Ltd.,Mumbai.All copyrights to this material vestswith IMS Learning Resources Pvt.Ltd.
No part of this materials either in part oras a whole shall be copied,printed,electronically reproduced,sold or distributed without the written
consent of IMS Learing Resources Pvt.Ltd.and any such violation would entail initiation of suitable legal proceedings.

Copyright

Login ID:8FA03376/Student Name:ANSHULGARG/Overall Score:28

1)

2)

3)

4)

5)

ite
d

1] dilutes
2] threatens
3] illuminates
4] enlightens
5] encompasses

iv
at
e

Li
m

Explanation:
The sentence relates the journey of the meteorite on earth to show something positive about a relationship stated after the last blank.
Thus, 'dilutes' and 'threatens' cannot be used. 'Enlightens' (i.e. 'imparts knowledge to') is completely unsuitable. 'Illuminates' (i.e. 'throws
light on') is a better fit than 'encompasses' (i.e. 'includes comprehensively'), as Black Beauty's story would shed light on the relationship
between meteorite hunters, collectors and scientists. Hence, [3].

3)

4)

5)

ou

2)

es

1)

rc

es

Pr

Question: 175
The following paragraph has some words missing. Choose the options that best fill the blanks.
1] social
2] synthetic
3] symbolic
4] symbiotic
5] semiotic

ar

ni

ng

Explanation:
The relationship has to be described by a term which speaks of a close association between the three groups mentioned, i.e., hunters,
collectors and scientists. It cannot be 'social' or 'synthetic' (artificial), as the groups can, at best, have a transactional relationship. There is
nothing 'symbolic' or 'semiotic' (relating to signs) about this relationship. 'Symbiotic', which indicates an interdependent relationship, fits
completely. Hence, [4].

Le

Question: 176
Mark the best substitute for the given phrase.

1)

A beginner or novice

2)

Ethos

3)

Neophyte

4)

Nostrum

5)

Milieu

IM

Novel

Explanation:
'Ethos' means 'the character or disposition of a community, group, person, etc'. 'Neophyte' means a 'beginner' or 'novice'. 'Nostrum' means

IMS Learning Resources Pvt.Ltd.,Mumbai.All copyrights to this material vestswith IMS Learning Resources Pvt.Ltd.
No part of this materials either in part oras a whole shall be copied,printed,electronically reproduced,sold or distributed without the written
consent of IMS Learing Resources Pvt.Ltd.and any such violation would entail initiation of suitable legal proceedings.

Copyright

Login ID:8FA03376/Student Name:ANSHULGARG/Overall Score:28

'a medicine sold with false or exaggerated claims with no demonstrable value'. 'Milieu' means 'surroundings, especially of a social or
cultural nature'. Hence, [3].

IM

Le

ar

ni

ng

es

ou

rc

es

Pr

iv
at
e

Li
m

ite
d

Question: 177
Read the following passage and answer the questions that follow.
Standing atop one of the many high-rises on the west bank of the Huangpu River, south of the iconic Bund waterfront area, it's possible to
get a clear view of the five-square-kilometre plot of land that Shanghai hopes will help showcase itself to the world.
In recent months, it's been almost impossible to read a news report from here that hasn't mentioned the fevered construction of pavilions
and facilities and the municipal reconstruction generally aimed at making the city's Expo the greatest show on earth.
In the decade since Shanghai won its bid to host the event, the city has morphed and grown in 2004, a maglev train started doing 8minute, 250 kilometre per hour laps to and from Shanghai Pudong International Airport; in March 2008 the airport itself fostered a second
terminal; early in 2010 the city's other airport, Hongqiao, did as well; and meanwhile the metro system has gained several new subway
lines.
'Expo' is short for 'Universal Exposition,' the inheritor of the World Fairs that began in London's Hyde Park, led by Queen Victoria's
husband, Prince Albert, in 1851. Traditionally, Expos have been about national pride (and in China's case, there's often plenty of pride to
salve, with the media and education system frequently reminding Chinese of the so-called hundred years of humiliation at the hands of
foreigners from the First Opium War until the end of World War II).
But while to the outside world the Shanghai Expo may be seen as an attempt, like the Beijing Olympics, to showcase China's emergence
onto the international stage, Shanghai's (slightly) smaller brother and two centuries' rival is in no mood to share in the festivities.
For while locals here are apt to point out that the Expo will draw an estimated ten times the number of visitors that the Beijing Olympics
did, the national leadership has resolutely refused to visit the site. And although Expo countdown clocks areubiquitousin Shanghai, none
are visible in Tiananmen Square, where previous events of national importance, such as the handovers of Hong Kong and Macau and the
Beijing Olympics, have traditionally been recognized.
Naysayers say Shanghai's official visitor number estimates are wildly inflated. But while it's true that Expos have historically lost money
some Expos actually exceed expectations. And given the general global fascination with China at present, as well as the country's sheer
size, Shanghai may end up beating its own forecasts.
So, if Shanghai does meet expectations, who are the likely winners and losers?
One clear winner of a successful Expo would be the government, with the army of bureaucrats in the Shanghai and central government
tied to the event able to claim some part of the credit (which will be officially described as a success whatever happens). Indeed, cynics
might also suggest that many of these bureaucrats will also benefit in more tangible ways, given the licenses and other approval
processes that must be secured by exhibitors, sponsors and vendors.
The Shanghainese, meanwhile, have benefitted from a host of infrastructure improvements and a general sprucing up of the area. Of
course they didn't have much choice in the matter. While in some countries electorates vote on whether or not to bid for an Expo based on
their own cost-benefit analysis of the situation, the decision to bid for the 2010 Expo was entirely in the hands of the Chinese government
(local and national). Their cost-benefit calculations are not audited, nor are they available for review.
According to a newspaper, the budget for the Shanghai Expo can be divided into three parts relocation, construction and operation. The
word relocation is, in some cases, something of a euphemism. Some Shanghainese have been forcibly moved to make way for the city's
facelift, not only from the five square kilometres of the site itself, but from other parts of the city that have launched Expo-driven
renovations.
But despite the construction mayhem and the ongoing loss of some architectural treasures and organic communities, many Shanghainese
view the Expo as a net win, with a dramatically improved transportation system, a cleaner city and a bright, internationalized new face to
present to the millions of likely foreign visitors.
Yet it won't just be Shanghai residents who will likely feel like they're benefiting. Mainland Chinese will generally see the event as the
government wishes them to as another milestone in China's 'peaceful rise' and feel proud regardless of whether they attend.
'In March 2008 the airport itself fostered a second terminal'. Which figure of speech is prevalent in this sentence?
1)

Hyperbole

2)

Metaphor

3)

Irony

IMS Learning Resources Pvt.Ltd.,Mumbai.All copyrights to this material vestswith IMS Learning Resources Pvt.Ltd.
No part of this materials either in part oras a whole shall be copied,printed,electronically reproduced,sold or distributed without the written
consent of IMS Learing Resources Pvt.Ltd.and any such violation would entail initiation of suitable legal proceedings.

Copyright

Login ID:8FA03376/Student Name:ANSHULGARG/Overall Score:28

4)

Personification

5)

Onomatopoeia

Explanation:
'Personification' is 'the attribution of human qualities to an object'. In the given sentence, the airport is said to have grown or raised another
terminal. Hence, [4].

ite
d

Question: 178
Read the following passage and answer the questions that follow.

omnipresent

2)

rare

3)

plentiful

4)

scattered

5)

colloquial

iv
at
e

1)

Li
m

What is the synonym of 'ubiquitous', as it is used in the passage?

Pr

Explanation:
Both 'ubiquitous' and 'omnipresent' mean 'existing everywhere, especially at the same time'. Hence, [1].

es

Question: 179
Read the following passage and answer the questions that follow.
What does the term 'organic communities' mean in the context of the passage?
Communities that grow organic food

2)

Communities that eat organic food

3)

Communities that have been forcibly uprooted

4)

Communities that showcased Shanghai's way of life

5)

Communities that had been a core component of Shanghai

es

ou

rc

1)

ni

ng

Explanation:
In the passage, the term 'organic communities' is used to represent the people that had been a core component of Shanghai and had
been living in the area for many years but were relocated to make way for the renovations to make the expo a grand success. Hence, [5].

ar

Question: 180
Read the following passage and answer the questions that follow.

Le

'The word relocation is, in some cases, something of a euphemism'. Which of the following correctly describes this sentence?
In some case, people that were made to move away were not paid relocation expenses.

2)

In some cases, people that were promised a home elsewhere were never allotted one.

3)

1)

4)

In some cases, people were forced to move away even against their wishes.

5)

In some cases, people protested against moving away.

IM

In some cases, people were given a choice to stay back too.

Explanation:
A euphemism is the substitution of a mild, indirect or vague expression for one thought to be offensive, harsh or blunt. So, according to
the author, the word 'relocation' is a mild term for what was actually going on. Only [4] correctly describes the quoted sentence. It is clearly
stated in the eleventh paragraph that some people were forcibly moved to carry out renovations for the expo. Hence, [4].

IMS Learning Resources Pvt.Ltd.,Mumbai.All copyrights to this material vestswith IMS Learning Resources Pvt.Ltd.
No part of this materials either in part oras a whole shall be copied,printed,electronically reproduced,sold or distributed without the written
consent of IMS Learing Resources Pvt.Ltd.and any such violation would entail initiation of suitable legal proceedings.

Copyright

Login ID:8FA03376/Student Name:ANSHULGARG/Overall Score:28

Question: 181
Read the following passage and answer the questions that follow.
Suggest a suitable title for the passage.
The Greatest Expo on Earth

2)

Problems Faced by the Shanghai Expo

3)

The Pros and Cons of Holding an Expo

4)

Shanghai's Makeover after the Expo

5)

The Truth Behind the Shanghai Expo

ite
d

1)

iv
at
e

Li
m

Explanation:
The passage does not describe the expo itself and 'greatest' is too positive a word to be used for describing the expo. Thus, [1] can be
eliminated. The passage doesn't describe any problems that the expo is facing or the pros and cons of holding an expo. Thus, [2] and [3]
can be eliminated. The passage describes Shanghai's makeover before the expo, not after it. [5] is a general title that could cover the
city's makeover and the insights into the likely winners and losers. Hence, [5].

While

3)

However

4)

Moreover

5)

Accordingly

rc

2)

ou

Despite

es

1)

es

Pr

Question: 182
The two statements given in each question can be combined into one by using an appropriate starting word or phrase. Choose the most
appropriate starter from the options.
S1: Communities have developed creative coping mechanisms to deal with possibleincoming deluges.
S2: Humanitarians and hydrological experts point to the perennial health concerns and safety hazards ofliving in flood plains beside rivers.

ng

Explanation:
Statement 1 describes actions taken by communities after considering what the experts have to say (in statement 2). [1] and [3] can be
eliminated as there is no contradiction. 'Accordingly' and 'moreover' cannot start the combined sentence. Hence, [2].

ar

ni

Question: 183
The two statements given in each question can be combined into one by using an appropriate starting word or phrase. Choose the most
appropriate starter from the options.

Le

S1: It has a far more important role in the Indian economy than its share of GDPsuggests.
S2: Agriculture's contribution to the GDP has been falling.
However

2)

When
Since

IM

3)

1)

4)

Although

5)

Before

Explanation:
The two sentences clearly contradict each other. 'However' can combine the two statements but it cannot start the combined sentence.
So, only 'although' correctly fits. Hence, [4].

IMS Learning Resources Pvt.Ltd.,Mumbai.All copyrights to this material vestswith IMS Learning Resources Pvt.Ltd.
No part of this materials either in part oras a whole shall be copied,printed,electronically reproduced,sold or distributed without the written
consent of IMS Learing Resources Pvt.Ltd.and any such violation would entail initiation of suitable legal proceedings.

Copyright

Login ID:8FA03376/Student Name:ANSHULGARG/Overall Score:28

Question: 184
The two statements given in each question can be combined into one by using an appropriate starting word or phrase. Choose the most
appropriate starter from the options.

When

2)

Since

3)

From the time

4)

Before

5)

Nonetheless

Li
m

1)

ite
d

S1: She found the room locked from the inside.


S2: Neena tried to open the door to get water for her ailing grandfather.

Explanation:
The two statements indicate actions taking place at the same time. Thus, only 'when' correctly combines the two statements. Hence, [1].

Even if

3)

Because

4)

If

5)

Yet

Pr

2)

rc

Even though

ou

1)

es

S1: This novel is full of passages that need to be interpreted and questions.
S2: I have decided to use this novel as a subject for my thesis.

iv
at
e

Question: 185
The two statements given in each question can be combined into one by using an appropriate starting word or phrase. Choose the most
appropriate starter from the options.

es

Explanation:
There seems to be no inherent contradictory or conditional relationship between the two sentences. One is likely to choose a novel as a
subject for a thesis 'because' it has scope for interpretation. Hence, [3].

ni

ng

Question: 186
The two statements given in each question can be combined into one by using an appropriate starting word or phrase. Choose the most
appropriate starter from the options.

Le

ar

S1: Some residents say the town is becoming 'more Indian'.


S2:Little Lhasa is home to about 12,000 Tibetans and permanent Western residents who together have placed their cultural stamp on the
town.
Although

2)

When
While

IM

3)

1)

4)

However

5)

Because

Explanation:
The two sentences contradict each other. So, only 'although' can begin the combined sentence correctly. 'However' can also combine the
two sentences but it cannot begin the combined sentence. 'Hence, [1].

IMS Learning Resources Pvt.Ltd.,Mumbai.All copyrights to this material vestswith IMS Learning Resources Pvt.Ltd.
No part of this materials either in part oras a whole shall be copied,printed,electronically reproduced,sold or distributed without the written
consent of IMS Learing Resources Pvt.Ltd.and any such violation would entail initiation of suitable legal proceedings.

Copyright

Login ID:8FA03376/Student Name:ANSHULGARG/Overall Score:28

Question: 187
The following paragraph has some words missing. Choose the options that best fill the blanks.
In studies on humans, researchers have found that people almost always choose the more expensive ___(27)___, regardless of whether
they know anything about it. In fact, when people are presented ___(28)___ two identical glasses of wine and told that one is more
expensive than the other, their brains show ___(29)___ activity in a reward-related area known as the medial orbitofrontal cortex (mOFC)
as they sip the more 'expensive' wine hinting that they actually perceive it as tasting better.
Results like these have made researchers ___(30)___ why we humans think this way. Do our ___(31)___ simply lead us to want
whatever's worth most or could it be that price is just one factor in a more complex value formula?

2)

3)

4)

5)

iv
at
e

Pr

1)

Li
m

ite
d

1]prize
2] piece
3]commodity
4]dividend
5] aid

es

Explanation:
The hint for the answer is in the second sentence, which mentions 'reward-related'. This hint makes [1] the best option. Hence, [1].

ou

rc

Question: 188
The following paragraph has some words missing. Choose the options that best fill the blanks.

3)

4)

5)

ng

ni

2)

ar

Le

1)

es

1] by
2] towards
3]with
4]over
5] to

IM

Explanation:
People cannot be presented 'to', 'towards' or 'over' glasses of wine. 'By' should be followed by a person. Thus, 'presented' should be
followed by 'with'. Hence, [3].
Question: 189
The following paragraph has some words missing. Choose the options that best fill the blanks.
1]plentiful
2] elevated
3]fine
4]sharp
5]hectic

IMS Learning Resources Pvt.Ltd.,Mumbai.All copyrights to this material vestswith IMS Learning Resources Pvt.Ltd.
No part of this materials either in part oras a whole shall be copied,printed,electronically reproduced,sold or distributed without the written
consent of IMS Learing Resources Pvt.Ltd.and any such violation would entail initiation of suitable legal proceedings.

Copyright

Login ID:8FA03376/Student Name:ANSHULGARG/Overall Score:28

1)

2)

3)

4)

5)

2)

3)

4)

5)

Li
m

Pr

es

1)

iv
at
e

Question: 190
The following paragraph has some words missing. Choose the options that best fill the blanks.
1]dread
2] wonder
3]worry
4]regard
5] guess

ite
d

Explanation:
In this sentence, the type of activity is being associated with 'better' perception. So, we need a positive qualitative word, as opposed to a
positive quantitative word. So, 'elevated', which means 'raised', is the best option. Hence, [2].

es

ou

rc

Explanation:
There is no indication that the correct word should be negative. So, 'dread' and 'worry' should be ruled out. 'Regard' does not fit
grammatically and researchers would not 'guess': they would analyse. 'Wonder' is the most appropriate choice and is supported by the
next sentence. Hence, [2].

Question: 191
The following paragraph has some words missing. Choose the options that best fill the blanks.

3)

ni
ar

2)

Le

4)

1)

ng

1]instincts
2] compulsions
3]analyses
4]values
5] egos

5)

IM

Explanation:
The answer has to be a word that is related to perception, as that leads to the choice. 'Compulsions' and 'values' cannot be the reasons
for choosing the most expensive wine. 'Analyses' is the opposite of the required word as the paragraph shows that the wine-sippers do not
think about the choices they are making. 'Instincts' is a better fit than 'egos' as the activity in the mOFC seems to be based on instincts
rather than egos. Hence, [1].

IMS Learning Resources Pvt.Ltd.,Mumbai.All copyrights to this material vestswith IMS Learning Resources Pvt.Ltd.
No part of this materials either in part oras a whole shall be copied,printed,electronically reproduced,sold or distributed without the written
consent of IMS Learing Resources Pvt.Ltd.and any such violation would entail initiation of suitable legal proceedings.

Copyright

Login ID:8FA03376/Student Name:ANSHULGARG/Overall Score:28

Question: 192
Mark the word that means the same as the capitalized word.

1)

Plenary

2)

Celestial

3)

Impalpable

4)

Verbose

5)

Complicated

ite
d

LOQUACIOUS

Li
m

Explanation:
'Loquacious' and 'verbose' mean 'characterized by excessive talk' or 'wordy'. 'Plenary' means 'full', 'entire' or 'absolute'. 'Celestial' means
'heavenly' or 'divine', and 'impalpable' means 'incapable of being perceived by the sense of touch' or 'difficult for the mind to grasp easily'.
Hence, [4].

iv
at
e

Question: 193
Mark the word that means the same as the capitalized word.

Voluble

3)

Abstruse

4)

Reticent

5)

Arcane

es

2)

rc

Garrulous

ou

1)

Pr

TACITURN

es

Explanation:
'Taciturn' and 'reticent' are synonyms, which mean 'inclined to silence' or 'reserved in speech'. 'Garrulous' and 'voluble' mean 'excessively
talkative'. 'Abstruse' and 'arcane' mean 'hard to understand'. Hence, [4].

ng

Question: 194
Mark the word that does not mean the same as the capitalized word.

2)

Prudent

3)

Meticulous

4)

Vigilant

5)

Churlish

ar

Cautious

Le

1)

ni

CIRCUMSPECT

IM

Explanation:
'Circumspect' means 'watchful' or 'cautious'. [1], [2], [3] and [4] are all synonyms of 'circumspect'. 'Churlish' means 'rude', 'mean' or 'difficult
to work with', so it is the right answer. Hence, [5].
Question: 195
Mark the word that does not mean the same as the capitalized word.
GRANDIOSE
1)

Pretentious

IMS Learning Resources Pvt.Ltd.,Mumbai.All copyrights to this material vestswith IMS Learning Resources Pvt.Ltd.
No part of this materials either in part oras a whole shall be copied,printed,electronically reproduced,sold or distributed without the written
consent of IMS Learing Resources Pvt.Ltd.and any such violation would entail initiation of suitable legal proceedings.

Copyright

Login ID:8FA03376/Student Name:ANSHULGARG/Overall Score:28

2)

Extravagant

3)

Flamboyant

4)

Ostentatious

5)

Ethereal

ite
d

Explanation:
[1], [2], [3] and [4] are all synonyms of 'grandiose', which means 'pompous', 'elaborate' or 'grand in an imposing or impressive way'. Only
'ethereal,' which means 'extremely delicate' or 'celestial', has a meaning different from that of the given word.Hence, [5].

ni

ng

es

ou

rc

es

Pr

iv
at
e

Li
m

Question: 196
Read the following passage and answer the questions that follow.
It was a crowning moment in Jeff Koons's long and extraordinary career. While Francis Bacon's $142m triptychdepicting Lucian Freud was
grabbing the headlines, at the same Christie's sale in New York, the man who has turned banality into high art was enjoying his own
triumph this week. He did it with one of his celebrated series of toy balloon dog sculptures " 10ft tall and constructed of stainless steel but
faithfully reproducing the contours and shiny plastic colours of a party balloon toy. It went for a staggering $58m (36m) " the highest sum
ever paid for a work by a living artist.
So what is it about Koons that has so captured the art world's imagination over the past three decades? He was avidly ambitious from a
young age; his early heroes were Salvador Dali, whom he met once while the Spanish surrealist was in residence in New York, and Andy
Warhol. From Warhol he borrowed the idea that, in a godless world saturated by media, art itself was just another commodity, albeit one
with extraordinary added value. And while Warhol made silk screen prints in his 'factory', Koons took the industrial metaphor much further,
employing dozens of skilled artisans to manufacture works such as the balloon dogs without any hands-on input from the artist himself.
Nearly three years ago, Koons threatened legal action against a bookshop in San Francisco selling bookends with a balloon dog motif. But
as a lawyer for the bookshop pointed out: 'As virtually any clown can attest, no one owns the idea of making a balloon dog.' Originality of
inspiration is not part of the Koons recipe: most of his work has its origins in pre-existing works of art or kitsch. But although critics often
use the words 'arid' and 'sterile' to describe his art, pieces such as the balloon dog have enduring fascination, and not only for the
collectors who watch their value rise and rise. The one exhibited for years outside Venice's Palazzo Grassi, overlooking the Grand Canal,
became one of the watery city's more endearing monuments. The artist himself claims to find emotional release through his work. 'I think
art takes you outside yourself, takes you past yourself,' he has said. 'I believe that my journey has really been to remove my own anxiety.'
Born in York, Pennsylvania, in 1955, Jeff Koons studied art at Maryland School of Art and in Chicago, idolized the Pop artist Ed Paschke
and became his studio assistant, then moved to New York where he worked on the membership desk of the Museum of Modern Art.
But then his career took a different turn. Instead of starving in a garret or waiting on restaurant tables while his talent flowered, he became
a Wall Street commodities trader. In contrast to the artist of popular imagination, the world of commerce was his natural home. Here was
an artist who felt no shame about making money but on the contrary made a profound study of what the contemporary artist needs to do
to become rich. And then he did it.

ar

According to the passage, Jeff Koons created art to:


make money while he was at the height of popularity.

2)

make art accessible to common people.

3)

experience emotional catharsis.

4)

follow in the footsteps of Dali and Warhol.

spite his worst critics.

IM

5)

Le

1)

Explanation:
Refer to the 3rdparagraph of the passage, where Jeff Koons gives 'emotional release' as his reason for creating art. Though he did want to
make money, [1] wrongly implies that he was already highly popular for some other reason and then took to creating art. The other options
are not implied at all in the passage. Hence, [3].
Question: 197
Read the following passage and answer the questions that follow.

IMS Learning Resources Pvt.Ltd.,Mumbai.All copyrights to this material vestswith IMS Learning Resources Pvt.Ltd.
No part of this materials either in part oras a whole shall be copied,printed,electronically reproduced,sold or distributed without the written
consent of IMS Learing Resources Pvt.Ltd.and any such violation would entail initiation of suitable legal proceedings.

Copyright

Login ID:8FA03376/Student Name:ANSHULGARG/Overall Score:28

The phrase 'industrial metaphor' refers to:


1)

the use of industrial material in art.

2)

the mass production of art.

3)

the mechanical use ofart.

4)

making the banal seem artistic.

5)

the tragedy of art.

Li
m

ite
d

Explanation:
Refer to the last sentence of the 2ndparagraph. The 'industrial metaphor' is used to suggest how art came to be mechanically reproduced
in great numbers, without any personal input from the artist. The same is implied about Andy Warhol's silk screen prints. Hence, [2].
Question: 198
Read the following passage and answer the questions that follow.

Lacking colour

2)

Lacking conviction

3)

Lacking creativity

4)

Lacking courage

5)

Lacking confidence

es

Pr

1)

iv
at
e

What does 'sterile' mean in the context of the passage?

ou

rc

Explanation:
Sterile means barren or lacking inspiration or vitality. In the context of the passage,sterile means lacking inspiration or creativity. [2],
[4] and [5] cannot be the meaningsof sterile and there is no implication in the passage that [1] is the answer.Hence, [3].

es

Question: 199
Read the following passage and answer the questions that follow.

The popularity of the toy balloon dog sculpture suggests:


a mockery of high art.

2)

society's obsession with material things.

3)

the overpowering effect of pop culture on people.

4)

people's canine longing for attention.

5)

people's fascination with toy merchandise.

ar

ni

ng

1)

IM

Le

Explanation:
The passage discusses how the toy balloon dog has caught the imagination of people the world over. In the third paragraph, the author
states that most of Koons' work has its origins in pre-existing works of art or kitsch (i.e. something of tawdry design, appearance or content
created to appeal to popular or undiscriminating taste). This points to [3] as the answer. Hence, [3].
Question: 200
Read the following passage and answer the questions that follow.
Which of the following words used in the passage means the same as 'outline'?
1)

Garret

2)

Motif

IMS Learning Resources Pvt.Ltd.,Mumbai.All copyrights to this material vestswith IMS Learning Resources Pvt.Ltd.
No part of this materials either in part oras a whole shall be copied,printed,electronically reproduced,sold or distributed without the written
consent of IMS Learing Resources Pvt.Ltd.and any such violation would entail initiation of suitable legal proceedings.

Copyright

Login ID:8FA03376/Student Name:ANSHULGARG/Overall Score:28

3)

Contour

4)

Headline

5)

None of the above

IM

Le

ar

ni

ng

es

ou

rc

es

Pr

iv
at
e

Li
m

ite
d

Explanation:
The word that means the same as 'outline' is 'contour' i.e. outline representing or bounding the shape or form of something. Hence, [3].

IMS Learning Resources Pvt.Ltd.,Mumbai.All copyrights to this material vestswith IMS Learning Resources Pvt.Ltd.
No part of this materials either in part oras a whole shall be copied,printed,electronically reproduced,sold or distributed without the written
consent of IMS Learing Resources Pvt.Ltd.and any such violation would entail initiation of suitable legal proceedings.

Copyright

You might also like